MasterDay2

A word from the author: MasterRQs and MasterDay2 are helpful only once you have basic knowledge of your subjects. Both of these files have the best and compile almost all radiographic images and cases available online. Extensive image coverage of each of topic has been done. Day 2 tests your basic knowledge of the subjects. I sincerely advise you to go through the mentioned topics properly- as the cases are mostly focussed on these. Know these diseases and medicines used, contraindications of them and MOA.

What to note/write on the paper provided in the exam ? You will be given 2 sheets to write on/if anything. Here is what you need to write. Draw a line in the centre of the first page, and write Generic name on one side and Trade name on another. During the exam in the first 4-5 cases they mention you both the class/Drug/generic name and the trade name eg :

Generic Trade Alendronate (Bisphosphonates) Fosamax Ethambutol (Anti TB) Abitol Zoledronic acid(Bisphosphonates) Reclast

By the time you are done with 4-5 cases they will skip the Generic names in few questions because they presume you should know them as they have been mentioned in the exam only. So If you are unaware of this, you will have to go back and look for the important information again question by question, but if u have made this small chart, it saves your time!

Must read topics 1. Hypertension and management 2. Myocardial infarction 3. Stroke 4. Diabetes 5. Syncope 6. Emergency management 7. NSAIDs 8. Antianxiety drugs 9. Osteoarthritis

May God be with us and help us nail NBDE PART 2  10. Alzheimer's disease 11. ADHD in child 12. Asthma 13. COPD 14. Bisphosphonates 15. Local Anesthesia and calculations 16. HIV 17. Hepatitis B 18. Opiods 19. Basal cell carcinoma picture 20. Papilloma picture 21. Prophylactic Antibiotics 22. Endodontic Diagnosis (Given in this file- Last section) 23. Practice Cases (Given in this file- Last Section)

1. Identify “AOT” On radiographs, the adenomatoid odontogenic tumor presents as a radiolucency (dark area) around an unerupted tooth extending past the cementoenamel junction.

It should be differentially diagnosed from a dentigerous cyst and the main difference is that the radiolucency in case of AOT extends apically beyond the cementoenamel junction.

Radiographs will exhibit faint flecks of radiopacities surrounded by a radiolucent zone.

It is sometimes misdiagnosed as a cyst. And frequently in anterior maxilla!

May God be with us and help us nail NBDE PART 2  2. Identify Dentigerous Cyst

3. Identify Eruption Cyst

4. Identify

Doubtful Answer Molar appears to have DD and Premolar – DI

May God be with us and help us nail NBDE PART 2  5. Identify? :

How to identify? See the outer boundary n shape. Compare it with adjacent teeth moreover i see it like a prepared tooth for crown to diagnose.

6. Identify Dentinogenesis imperfecta

May God be with us and help us nail NBDE PART 2 

7. Identify External Resorption

8. Identify Genial tubercle

May God be with us and help us nail NBDE PART 2 

9. Identify Mandibular Canal

May God be with us and help us nail NBDE PART 2  10. Identify Ameloblastoma

11. Identify Fusion

May God be with us and help us nail NBDE PART 2  12. Identify Lateral Periodontal Cyst

13. Identify Complex Odontoma

It's a complex odontome. Compound would show enamel dentin and cementum separately.

May God be with us and help us nail NBDE PART 2 

14. Identify Cementoblastoma

15. Identify Underexposed / Over Fixed

May God be with us and help us nail NBDE PART 2  16. Identify Intermaxillary Suture

17. Identify Zygomatic Process

May God be with us and help us nail NBDE PART 2 

18. Identify Coronoid Process

19.

May God be with us and help us nail NBDE PART 2  20.

21. Identify

May God be with us and help us nail NBDE PART 2 

22. Identify

23. Identify Stafne’s bone Cyst

May God be with us and help us nail NBDE PART 2 

24. Identify Recurrent Caries:

May God be with us and help us nail NBDE PART 2  25. Identify Age and Diagnosis?

Over 7 Years Diagnosis would be retained deciduous lower incisors and coming to age is would be 7-8 here we have to consider the development of roots of 6 and roots of upper central incisors and lower lateral incisors. Coming to loss of D in 3rd arch, ref; Mc Donald pediatric book, if any primary teeth lost before 7 yr there would be delayed eruption of permanent teeth and if loss of primary teeth after 7 yrs would lead to premature eruption of permanent teeth. Here premolar is in eruption stage so it should be considered too. So age would be 7-8 yrs.

26. Learn the fact

May God be with us and help us nail NBDE PART 2  Complete craniofacial dysjunction by the Le Fort III osteotomy allows the surgeon to alter the orbital position and volume, zygomatic projection, position of the nasal root, frontonasal angle, and position of the maxilla and to lengthen the nose. The Le Fort II osteotomy allows the surgeon to alter the nasomaxillary projection without altering the orbital volume and zygomatic projection. The Le Fort I osteotomy allows for correction primarily at the occlusal level affecting the upper lip position, nasal tip and alar base region, and the columella labial angle without altering the orbitozygomatic region

27. The appearance of the curve of spee on the panoramic suggests that the patients chin was positioned too: A. Far forward B. Far back C. High D. Low

Answer is HIGH

28. No answer available

May be lingual tori

29. Identify Underexposed/Over fixed

May God be with us and help us nail NBDE PART 2  +Lower border of mx.sinus

30. Identify gemination, fusion has just two roots

May God be with us and help us nail NBDE PART 2  31. Identify Answer is below the picture

32. Identify flap Semilunar Flap

May God be with us and help us nail NBDE PART 2  33. Identify Answer is below the Picture

May God be with us and help us nail NBDE PART 2 

34. Identify Answer is below the picture

May God be with us and help us nail NBDE PART 2  35. Identify Answer is below the picture

May God be with us and help us nail NBDE PART 2  36. Identify

we have retained primary 1ts molar in lower right side Missing lower 3rd molar bilateral or extracted Chin is up means  frown Chin is down means  smile

In this Pano ... Chin down smiling Pt is 23 years’ old

37. Pt adult male. Present asymptomatic, necrotic, ulcerated area involving palatal mucosa. Histologically presents lobular necrosis of glandular parenchyma with squamous metaplasia and hyperplasia of ductal epithelium. Dx & Tx?

Necrotizing sialometaplasia. it is caused by ischemic necrosis of minor salivary glands. it will heal in 6-10 wks.

May God be with us and help us nail NBDE PART 2 

38. Identify 8 is angle of Mandible and 9 is Hyoid Bone

39. Identify

Peutz-Jeghers Syndrome. Intraorally pigmentations may be located anywhere in the mucosa. Most frequently hard palate, Buccal mucosa and gingiva

May God be with us and help us nail NBDE PART 2 

40. Identify

1- middle cranial fossa 5- styloid process 17- ear

May God be with us and help us nail NBDE PART 2 

41. Identify

Dens in Dente

May God be with us and help us nail NBDE PART 2  42. Identify 10- glenoid fossa 11- articular eminence 12- condyle 13-vertebra

43. Identify : Compare with Dens in dente : 42nd Question

May God be with us and help us nail NBDE PART 2  44. Identify 2-orbit 3- zygomatic arch 4- hard palate 6- max tuberosity

45. Identify

Eagle’s Syndrome Elongated Styloid and calcification of stylohyoid ligaments.

May God be with us and help us nail NBDE PART 2  46. Identify

odontogenic myxoma: non- encapsulated, no resorption of teeth, cortical expansion, honey- comb appearance

47. Identify Punched out lesions without sclerotic border...multiple myeloma Punched out lesions with or without sclerotic border.... Langerhans cell histiocytosis

May God be with us and help us nail NBDE PART 2  48. Identify

The arrow is pointing to alternating RLs at the level of border of mandible. This is also known as "onion skin" and it's seen in GARRE's Osteomyelitis.

49. Identify – DAY 1 Y is formed between nasal floor and maxillary sinus Y line of Ennis The straight line resembles the basal floor and the curved one for the maxillary sinus.

May God be with us and help us nail NBDE PART 2  50. Identify

Answer: Antral pseudocyst (mucous retention pseudocyst)

51. Identify

May God be with us and help us nail NBDE PART 2  52. Identify Internal Resorption

53. Turner's Hypoplasia***-Frequent pattern of enamel defects seen in permanent teeth secondary to periapical inflammatory disease of the overlying deciduous tooth. -The altered tooth is called Turner's tooth.

May God be with us and help us nail NBDE PART 2  54. Characteristics of enamel defects in Turner's Hypoplasia -Vary from focal areas of white, yellow or brown discoloration to extensive hypoplasia which can involve the entire crown. Most frequently noted in permanent bicuspids because of their relationship to the overlying deciduous molars.

55. In regards to traumatic injury, which teeth are most affected by Turner's Hypoplasia Maxillary central incisors

56. Identify problem, cause, treatment clinical features and treatment -Ingestion of fluoride results in enamel defects called dental fluorosis. -Permanent hypomutation of the enamel in which there is an increased surface and subsurface porosity of the enamel. -Altered tooth structure presents as areas of lusterless, white opaque enamel with zones of yellow to dark-brown discoloration. treatment: composites and full crown coverage

May God be with us and help us nail NBDE PART 2  57. Critical years for dental fluorosis During the 2nd and third years of life when anterior teeth are forming and fluoride levels are greater than 1 pt per million ingested.

58. definition*** Loss of tooth structure caused by tooth to tooth contact during occlusion and mastication Destruction accelerated by: 1) poor quality or absent enamel 2) Premature contacts, edge to edge occlusion 3) intraoral abrasives, erosion or grinding habit

59. Abrasion definition Loss of tooth structure secondary to the action of an external agent. Most common source is tooth brushing.

May God be with us and help us nail NBDE PART 2 

60. Erosion definition Loss of tooth structure caused by chemical reaction (other than bacteria) -usually to an acid external: lemon, vinegar, soft drinks internal: gastric secretions such as perimylolysis (people suffering from anorexia or buliema present this)

61. Attrition most often seen in these teeth and specific surfaces -Deciduous and permanent teeth -Occlusal and Incisor region of maxillary and mandibular teeth -Lingual of Maxillary anteriors -Labial of mandibular anteriors

62. Abrasion

May God be with us and help us nail NBDE PART 2 

63. Internal Resorption – Day 1

Destruction of teeth accomplished by cells located in the dental pulp.-Rare occurence, when injury to pupal tissue occurs such as physical trauma or pulpitis. -Process continues as long as vital pulp tissue is present. -Uniform well circumscribed symmetric radiolucent enlargement of the pulp chamber or canal. Pinktooth of Mummery. -When it affects the coronal pulp and expands it, the crown can display a pink discoloration:

64. External Resorption*** -Destruction of teeth accomplished by cells in the periodontal ligament -Extremely common, all patients likely to have root resorption on one or more teeth. -impacted teeth can hit other teeth and cause damage. -Presents as a "moth-eaten loss of tooth structure in which radiolucency is well less-defined. -Most cases involve the apical or midportions of the tooth.

65. Environmental Discoloration of teeth: Extrinsic Staining examples 1) Bacterial Stains: cause surface staining of enamel, dentin and cementum. Seen initially on labial surfaces of the maxillary anterior teeth in the gingival 1/3. 2) Tobacco products, tea, coffee: brown discoloration of the surface enamel. On lingual surface of anterior teeth but usually widespread and less intense. 3) Medications: Fluoride and chlorhexidine or many other oral antiseptics like Listerine.

May God be with us and help us nail NBDE PART 2  66. Environmental Discoloration of teeth: Intrinsic Staining Examples: 1) Congenital erythropoietic polyphyria (Gunther's disease) -Autosomal recessive disorder of porphyrin metabolism -results in increased synthesis and excretion of porphyrins and related precursors. -Diffuse discoloration of the dentition is noted as a result of the deposition of porphyrin in the teeth. -Stains teeth red -eyes are blood shot, show hirutism, sensitive to light, "wear- wolf disease"

67. Intrinsic staining examples: 2) Hyperbilirubinemia -During this syndrome, developing teeth may also accumulate pigment and become intrinsically stained. -Gallbladder closes, causes bile to accumulate in blood and cause brown staining in system. -Deciduous teeth are affected as a result of this syndrome during the neonatal period. -teeth extremely dark brown Two most common causes: erythroblastosis fetalis biliary atresia

68. Primary impaction of deciduous teeth is rare but when it occurs it affects this tooth*** second molars.

May God be with us and help us nail NBDE PART 2 

69. Ankylosis plays an important role in pathogenesis of impacted teeth. Permanent teeth most affected by impaction are*** third molars (mand then max) and then maxillary cuspids. Treatment for impacted teeth include: orthodontics, long-term watch, surgical removal, assisted eruption and transplantation.

Ankylosis definition, clinical features and treatment Cessation of eruption after emergence of teeth -occurs at any age but most obvious during first 2 decades of life: 9-18 yrs of age -occlusal plane of involved tooth is below that of adjacent teeth. -A sharp solid sound on percussion of the involved tooth noted if more than 20% of root is fused to the bone. Absence of periodontal ligament space may be noted. -fusion of root into jaw bone is smooth (resorption would show up ragged on radiograph) Treatment: Fused to adjacent bone, don’t respond to ortho, primary molars best treated with extractions and space maintainers.

70. lack of development of one or more teeth Uncommon in deciduous dentition. Very common in permanent dentition (pic example of hypodontia during ectodermal dysplasia)

71. and mesiodens also increased number of teeth and additional teeth are called supernumerary

May God be with us and help us nail NBDE PART 2  72. Clinical features of Hypodontia (teeth it most frequently occurs with) (most-> least) 1) third molars 2) second premolars 3) lateral incisors -More common in females than males

Hyperdontia clinical features -Single tooth hyperdontia occurs more frequently in the permanent dentition and 90% present in the maxilla anterior incisor region, followed by maxillary fourth molars and mandibular fourth molars.

73. Multiple supernumerary teeth (nonsyndromic) occur most frequently in the: mandibular premolar region. The differential diagnosis for multiple supernumerary teeth are 1) Gardner's syndrome (malignancies in large intestine) 2) Cleidocranial dysplasia

74. Natal teeth teeth present in newborns -should be extracted or will cause child severe pain

75. teeth present within the first 30 days of life (represent pre-maturely erupted deciduous teeth not supernumerary teeth) -should be extracted or will cause child severe pain

May God be with us and help us nail NBDE PART 2  76. Germination -A single enlarged tooth or jointed tooth in which the tooth count is normal when the anomalous tooth is counted as one. -1 crown, bifid pulp chamber, 1 root and 1 pulp canal. -Both primary and permanent dentitions -High frequency in the anterior maxillary region

77. Fusion -A single enlarged tooth or joined (double) tooth in which the tooth count reveals a missing tooth when the anomalous tooth is counted as one. -2 crowns, 2 separate pulp chambers, and 1 root. -Both primary and permanent dentitions -High frequency in the anterior maxillary region

78. -union of two adjacent teeth by cementum alone without the confluence of the underlying dentin. -Presents as two fully formed teeth, jointed along the root surfaces by cementum. -Frequently in the posterior maxillary region.

May God be with us and help us nail NBDE PART 2  79. Accessory Cusps: -Seen in oriental groups -Cusp-like elevation of enamel located in central groove or lingual ridge of the buccal cusp of permanent premolar or molar teeth. -Usually bilateral, marked by mandibular premolar predominance. -results in occlusal problems and prone to fracture frequently resulting in pupal exposure because the dens evaginatus contains pupal tissue.

80. **** -Deep surface invagination of the crown or root that is lined by enamel. -Due to defect in enamel -Two forms: coronal or radicular -Coronal is seen more frequently -Teeth affected mostly: permanent maxillary lateral incisors and maxillary central incisors -Invagination maybe large and resemble a tooth within a tooth "dens in dente" -If opening of invagination becomes carious, pupal necrosis may result. Treatment: small restoration

81. Ectopic Enamel presence of enamel in unusual locations, mainly the tooth root. (if u have enamel bud at junction of CEJ...periodontal defect will result) ex) enamel pearls -hemispherical structures of enamel -most frequent: roots of max. molars, mand molars: second most frequent. -majority at furcation area or near CEJ. -precludes normal periodontal attachment. -less resistant to breakdown, rapid loss of attachment likely.

May God be with us and help us nail NBDE PART 2 

82. *** Enlargement of the body and pulp chamber of multi-rooted tooth with apical displacement of pulp chamber and bifurcation. -affected teeth are rectangular in shape -pulp chambers have increased apico-occlusal height and bifurcation (if present) close to apex. -Unilateral or bilateral -Permanent teeth affected more -Isolated trait or w/ 1) Down's 2) Klinefelter's 3) Ectodermal dysplasia 4) Amelogenesis imperfecta

83. *** Abnormal angulation or bend in the root, or less frequently the crown of a tooth. -arise following an injury that displaces that calcified portion of the tooth germ with the remainder of the tooth being formed at an abnormal angle. -Permanent maxillary incisors most frequently affected.

84. Supernumerary roots*** increased number of roots on a tooth. -occurs more frequently on third molar teeth.

85. Hypoplastic Type Generalized: pinpoint pits Localized: horizontal row of pits in the middle 3rd of facial surface Incisal edge or occlusal surface= not affected

May God be with us and help us nail NBDE PART 2  Crown prep: like teeth on x-ray! <- amelogenesis Differentiation

86. Hypocalcified -Matrix laid down normally, abnormal calcification/mineralization takes place -SOFT ENAMEL EASILY LOST -YELLOW-BROWN OR ORANGE ENAMEL -EASILY STAINED, RAPID CALCULUS FORMATION X: ray: density of enamel similar to dentin

87. Hypomaturation type Matrix lad down, mineralizes normally, but crystal DO NOT form -teeth normal in shape, OPAQUE, WHITE-BROWN-YELLOW DISCOLORATION -Enamel is SOFT AND CHIPS OFF, pierced with explorer x-ray: enamel looks like dentin (enamel has fallen off DEJ)

88. Ameleogenesis imperfecta picture no enamel on teeth

May God be with us and help us nail NBDE PART 2  89. Dentinogenesis Imperfecta** -Hereditary disturbance in formation of dentin -aka "hereditary opalescent dentin" -May be seen w/ association to osteogenesis imperfecta -most cases in US traced from whites from English Channel

pic: large bulbous crowns and thinning roots

90. Dentinogenesis Imperfecta clinical description** -Autosomal dominant -Severity depends on age *DECIDUOUS most severely affected -Followed by perm. incisors and 1st molars (order of eruption) -2nd and 3rd molars LEAST AFFECTED -Exhibit a blue/brown discoloration with translucence -Enamel strips off easily from underlying dentin (like amelogenesis!) **Differentiation: Bulbous crowns (like tulips!), cervical constriction towards roots, thin roots, obliteration of root canals and pulp chambers. "shell teeth"= LARGE PULP CHAMBERS, THIN DENTIN

91. Dentinogenesis Imperfecta broad crowns, thin roots, cervical constriction, pulp chamber disappear/non-existant, enamel present but easily taken off (DEJ doesn't hold enamel well)

92. ** (Type 1) Type 1 or radicular dentin dysplasia (rootless teeth) -short roots: malformation of root dentin -Autosomal dominant pattern -Enamel and Coronal dentin =NORMAL *x-ray: deci teeth SERVERLY affected with little or no pulp and very short roots. *perm teeth: little to no pulp present to CRESCENT SHAPED PULP CHAMBERS.

May God be with us and help us nail NBDE PART 2 

93. Dentin Dysplasia (Type 2)** CORONAL Dentin Dysplasia -Similar to DI!!!!! -Autosomal dominant -Blue/brown translucency of teeth X-ray: bulbous crown, cervical constriction, thin roots, early obliteration of pulp chamber Perm Teeth: classic thristle-tube or flame shape CANT TELL DIFFERENCE BETWEEN DD TYPE 2 AND DI!

94. "ghost teeth" -uncommon condition found in both dentitions -localized non-hereditary developmental abnormality involving enamel, dentin and pulp. (ALL LAYERS AFFECTED) -idiopathic -no racial or sexual predilection bi-modal peak: eruption of prim teeth 2-4 years perm dentition: 7-11 yrs

95. Regional Odontodysplasia Clinical Characteristics -Several contiguous teeth affected usually in maxillary anterior -Involvement of primary teeth is followed by perm teeth being affected -failure of eruption -teeth that erupt have small irregular, brown - yellow rough surfaced crowns x-ray: -thin enamel and thin dentin surrounding enlarged pulp -pale wispy image of teeth "ghost teeth" -densities of enamel and dentin appear fuzzy -teeth have no shape -localized

May God be with us and help us nail NBDE PART 2  96. Congenital leukokeratosis () It is a rare disorder which is inherited in an autosomal dominant trait.

Symptoms and signs: present as white, whitish-greyish, peeling, it is a painless and complication-free disorder on the buccal mucosa and the mucosal layers of the nose, oesophagus and the ano-genital region. These lesions appear first at the time of birth or during childhood. This disorder occurs mostly on the loose mucous membrane of the mouth (buccal mucosa, soft palate, the ventral surface of the tongue, the mucosal surface of the lips, floor of the mouth, etc.). The gingival and the dorsal surfaces of the tongue are usually unaffected.

Differential diagnosis: leukoplakia, , lichen oris, fungal infections.

97. Mechanical injuries

The main cause of traumatic ulcers is usually a single physical damage (for example, fights, sports, epilepsy, bone, sharp, broken tooth and prosthesis, etc.), which causes yellowish, painful, soft changes various in diameter on the affected mucosal tissue. These ulcers are surrounded by a thin, erythematous halo. They heal spontaneously in 6–10 days but leave scarring behind.

Differential diagnosis: specific ulcers (tuberculosis, lues), aphtha or carcinoma.

May God be with us and help us nail NBDE PART 2  98. fissuratum (denture granuloma)

There are two major factors in the development of a denture granuloma: the pathologically mobile denture and the pressure of its edge on the surrounding mucosal tissue. An may most commonly occur in the middle third of the mandibular bones and is more common on the buccal surface than on the lingual surface. The epidermal layer of the mucosa may develop increased keratinisation. These lesions may become pale, hard and more flexible to the touch where the accumulation of connective tissue fibres is more extensive.

Differential diagnosis: gingival cancer, Hydantoin hyperplasia, epulis.

99. Morsicatio buccarum et labiorum The cause is that nervous, anxious children and young adults may repeatedly chew or bite their facial mucosa, lips or sometimes even their tongue as a bad habit. This constant mechanical trauma causes white desquamation of the effected epithelium, with erosions or even ulcers to develop mainly in the line of dental occlusion and buccal mucosa. This bad habit is commonly accompanied by bruxism.

Differential diagnosis: leukoplakia, candidiasis, ulcers caused by inflammations and cancer.

May God be with us and help us nail NBDE PART 2  100. Recurrent infection (herpes labialis, cold sore) Pathogen: Herpes simplex virus After the primary infection, the virus becomes latent in the epithelium and in the ganglia of the trigeminal nerve.

Symptoms: it usually affects the lips or the boundary between the lips and the skin. Predisposing factors include exposure to sun, fever (fever blisters), upper respiratory tract infections, pneumonia, malaria, meningitis, physical or emotional stress, menstruation, dental treatment, stress, gastroenteritis, AIDS, pregnancy, trauma, local irritation, etc. Before the development of blisters, fever and enlargement of lymph glands may develop. At the onset of the disease an itchy, burning sensation (prodromal) and redness are present. A few hours later cluster of vesicles are formed, which are 1-3 millimeters in diameter. Vesicles become coalesced, and after two or three days they rupture and become ulcerated with yellowish crusts. The surrounding skin is erythematous. Skin lesions usually heal in 8-10 days without scarring. Sites and frequency of recurrences vary from patient to patient.

Differential diagnosis: Herpes zoster, .

101. Verruca vulgaris (common dentino)

Pathogen: human papillomavirus (HPV) – a member of the papovavirus family. The wart occurs less frequently in the oral cavity than on the skin, but more commonly than previously believed.

Symptoms: they usually appear on the lips, tongue and palate, but any other location in the oral cavity may occur. They appear abruptly and grow rapidly (viral origin), they are broad based and have a whitish- grey, papillary surface (usually feel hard on the skin). Verrucae may appear as multiple and confluent lesions.

Differential diagnosis: Papilloma, verrucous leukoplakia

May God be with us and help us nail NBDE PART 2  102. Acute pseudomembranous candidiasis (thrush) The commonest Candida albicans- caused oral disease.

Predisposing factors: inappropriate oral hygiene, heavy smoking, xerostomia, radiotherapy, the administration of antibiotics, corticosteroids or cytostatic drugs, diabetes mellitus, chronic systemic diseases, autoimmune diseases, leukemia, AIDS, immunosuppression. It mostly presents in newborns and children suffering from chronic illnesses.

Symptoms: the appearance of cream-coloured or pearl-white, bluish-white plaques which leave a slightly burning, erythematous surface if wiped off is characteristic of this type of candidiasis. Predilection sites include the buccal, the palate and the tongue. It is common in newborns and infants. In case of acute pseudomembranous candidiasis in adults, an underlying systemic disease has to be considered.

Differential diagnosis: Lichen, leukoplakia (cannot be wiped off), morsicatio buccarum, allergic .

103. Chronic atrophic candidiasis (denture stomatitis) It is a common chronic disease of the mucous membrane especially in patients wearing upper dentures. Tight fitting dentures provide favorable circumstances: they practically function as a substrate for the growth of Candida.

Symptoms: well-circumscribed, red, edematous, sometimes eroded mucous membrane corresponding with the plate of the upper denture is characteristic of the disease. Chronic atrophic candidiasis is symptom-free. Differential diagnosis: contact allergy of the palate.

May God be with us and help us nail NBDE PART 2  104. Recurrent aphthous ulcer (Mikulicz’s aphtha; minor aphtha)

The etiology is not yet explained. It might be an autoimmune, viral, bacterial (Streptococci), genetic (familial) or allergic disease or could be influenced by mechanical injuries, gastrointestinal diseases (Crohn’s disease), psychic stress, menstrual cycle (hormonal factor), walnut or chocolate consumption, etc., or by hypovitaminosis (B12, folic acid).

It is the most frequent disease of the oral mucous membrane. Recurrent aphthae are more frequent in adulthood and in women. It is rare among smokers (mucosal hyperkeratinisation).

Symptoms: initially erythema and oedema develops on the corresponding area, accompanied by a nipping, burning sensation or by paraesthesia. It can be either solitary or multiple. The development of ulcers is not preceded by vesicle formation. Well-demarcated lenticular (3–10 mm) painful ulcers on erythematous bases, covered with yellowish fibrous pseudomembranes are present. After 1 or 3 weeks, they heal without leaving scars. They might persist for years, but their frequency varies from person to person. They might be accompanied by lymphadenomegaly, too. Predisposed areas are the vestibular surface of the lips, the floor of the mouth, the ventral surface of the tongue and the cheek (non-keratinized mucosal epithelium). It is rare on the hard palate, the gingiva and the back of the tongue; these areas are usually affected by gingivostomatitis herpetica.

Differential diagnosis: herpetic stomatitis, allergic stomatitis.

105. Major aphtha (Sutton’s aphtha) Recently major aphthae have been considered to be the rare severe form of aphthous ulcer.

Symptoms: 1–4 cm in diameter, usually solitary, but might have multiple forms, too (giant aphtha). Very painful crater-like ulcers of variable depth, covered with yellowish-grey fibrous pseudomembranes. The lesion usually heals with deforming mucosal scarring after epithelization. Women are usually more affected. Predilection areas are the oropharyngeal region, the soft palate, the cheeks and the tongue. It causes severe pain, fever, ,

May God be with us and help us nail NBDE PART 2  lymphadenitis and difficulties in swallowing or eating. It may persist for several months or even for one or two years and ends to recur. Biopsy is mandatory Differential diagnosis: tumorous ulcer, decubital ulcer, specific ulcer: TB lesions

106. Pemphigus vulgaris Pemphigus vulgaris is a chronic, life- threatening autoimmune skin, mouth and other mucous membrane disorder characterized by blistering of the skin and . Sometimes pemphigus appears in reaction to a viral infection, certain drugs (D-penicillamine, antihypertensive Captopril), thymoma or myasthenia gravis. Symptoms: it usually affects elderly women. Blistering starts intraepithelially, the surrounding mucosa is not erythematous. The thin-walled intraoral bullae easily burst leaving painful erosions covered with whitish-grey fibrinous diphtheroid coating. The erosions may also bleed. Painful erosions may cause difficulty swallowing and eating and increased salivation. Erosions heal without scarring. Sites of greatest involvement include the buccal mucosa, tongue, palates and the gingiva. Nikolsky’s sign is positive (when the surface of the unaffected skin and oral mucosa is rubbed, and the skin and mucosa separate easily). The diagnosis is established on the basis of skin symptoms and the histopathological examination. In order to adjust the drug therapy, hospitalization is necessary. Differential diagnosis: herpetic gingivostomatitis, erosive lichen oris, .

107. Varicosity of the tongue It is a diffuse dilation of the veins occurring in an elderly age (varix). The dark blue varicosity on the ventral surface of the tongue or sometimes on the floor of the mouth is often called caviar lesion. The varicosity of the tongue is most common among patients suffering from cardiopulmonary diseases. Due to its tumour-like surface, it may cause cancerophobia.

Differential diagnosis: haemangioma, Kaposi’s sarcoma, malignant melanoma.

May God be with us and help us nail NBDE PART 2  108. Angular (perlèche) In children and young adults Staphylococcus or Streptococcus infection results in pyoderma of the labial commissure.

109. Median rhomboid The disease used to be classified as a developmental disorder (the persistence of tuberculum impar), but today the role of chronic Candida infection and decreased vascularity is emphasized. It is common among smoking men and among diabetic patients. The lesion occurs more frequently in denture-wearing patients.

May God be with us and help us nail NBDE PART 2 

110. The cause of the disease is not clarified; familial predisposition, psychological factors, vitamin deficiency, malnutrition, exudative diatheses and endocrine disorders may play a role in the development of the lesion t can be an accompanying symptom of psoriasis, reactive arthritis and diabetes mellitus.

111. Lingua villosa (hairy tongue) The condition is considered to be a multi-aetiological disease, which presents more frequently in elderly denture-wearing patients. Hairy tongue is often idiopathic, but it can also be triggered by antibiotics, corticosteroids, vitamin A or B deficiency, radiotherapy, chemotherapy, inappropriate oral hygiene, emotional stress, Candida albicans, heavy smoking, gastrointestinal disorders or hydrogen peroxide.

Symptoms: the hairy surface is caused by the elongation, thickening (hypertrophy) and keratosis of the filiform papillae. The yellowish-greenish, brownish-black colour can be explained by the presence of pigment-producing bacteria and Aspergillus species (not Candida albicans), foods, medications, consumer goods and smoking.

Differential diagnosis: Coated tongue.

May God be with us and help us nail NBDE PART 2  112. Xerostomia

Xerostomia is not a disease but a symptom. Xerostomia may be caused by a wide range of conditions, the main cause being temporary or permanent cessation of salivary secretion.

113. Patient with due to mouth breathing

114. Pregnancy epulis (If given pt is pregnant otherwise Epulis

May God be with us and help us nail NBDE PART 2  115. Ulcers due to agranulocytosis

116. due to calcium- channel blocker medication

117. Periapical abscess - fistula with opening

May God be with us and help us nail NBDE PART 2  118. A lateral periodontal cyst can also show the signs of a periodontal abscess. In case of infection, there is shiny, reddish swelling besides the root, and a fistula may often develop. This disease has its typical radiographic appearance: sharp edged, radiolucent picture, wide periodontal space around the root.

119. Aggressive periodontitis is a term including Local Juvenile Periodontitis and General Juvenile Periodontitis, also known as Early Onset Periodontitis (EOP).

Typical features are fast progression and vertical bone resorption starting proximally, resulting in intra-osseal periodontal pockets. Due to the fast progression and destructive manner, secondary symptoms (hypermobility, pocket formation, abscess formation, changes in position and angulation, hyper eruption) develop quickly. After thorough questioning, the presence of the disease in the family is often revealed.

Secondary signs of AgP: tipping, movement, diastema forming between central incisors, deep bite

May God be with us and help us nail NBDE PART 2  120. Peri-implant mucositis: reversible soft tissue lesion around implants

Peri-implantitis: inflammation affecting osseointegrated implants in function, resulting in supporting bone loss

121. Papilloma – Cauliflower appearance

Papilloma is a benign epithelial tumour, which frequently appears on the oral mucosa. Local irritation and infection, especially viral infection (human papilloma virus) may play a role in its development.

Symptoms: a pedunculated or sessile, white, or whitish-grey, papillary surfaced tissue growth, which does not cause any symptoms. A slowly proliferating, smaller or larger epithelial tumour may also develop. It occurs mostly in the palates, tongue, gingiva and lips.

Differential diagnosis: verruca vulgaris, condyloma acuminatum and .

May God be with us and help us nail NBDE PART 2 

122. Fibroma Fibromas are benign, slow-growing tumours of the oral cavity composed of connective tissue. Fibroma is the most common tumour of the oral mucosa, which particularly occurs on the buccal mucosa and on the tongue. In the oral cavity, fibromas are reactive proliferative lesions, which develop secondarily to irritation (by sharp tooth edges, artificial teeth or calculus) or low grade infection. The lesion feels solid or soft on palpation. It is sessile or pedunculated, smooth-surfaced, reddish or pale and covered with intact mucosa, but as a result of mechanical irritation it may become whitish leukoplakia due to keratinization or may become ulcerated. No other clinical sign or pain can be observed.

Differential diagnosis: verruca vulgaris, condyloma acuminatum and verrucous carcinoma. epulis, , granuloma fissuratum and neurinoma.

123. Cavernous haemangioma This type of haemangioma is a hamartoma (not a neoplasm), which histopathologically does not have a capsule. It mostly involves the tongue, lips and the buccal mucosa, and reaches an extreme size (macrocheilia, macroglossia), thus hindering speech and swallowing. It occurs as a flat or exophytic, painless growth with a raspberry-like appearance. Its colour ranges from red to dark blue

May God be with us and help us nail NBDE PART 2  124. Pyogenic granuloma The tumour-like reactive tissue proliferation may develop in response to trauma or infection, but it may also occur as a result of hormonal changes (pregnancy, menopause, or puberty), the administration of oral contraceptives, steroid drugs and anti-diabetics. Most frequently it develops on the gingiva, bucca, lips and tongue. It is a raised, pedunculated, flat or wart- like lesion with a dark red or pink colour depending on the vascularisation of the affected area. In case of ulceration, it is covered with yellowish fibrin. Its diameter ranges from a few millimetres to a centimetre. It is a painless tumour, which easily bleeds.

125. Postextraction granuloma This reactive, inflammatory tumour-like lesion develops after tooth extraction as a result of retained root, broken pieces of bones, amalgam or other foreign body.

May God be with us and help us nail NBDE PART 2  126. Epulis granulomatosa It is not a real neoplasm, but a reparative tissue proliferation arising from the mucoperiosteum of the tooth socket or from the periodontal membrane. The diagnosis of ”epulis” may be applied only clinically. The growth is a painless, red lesion, which is prone to bleeding and occurs mostly on the vestibular surface of the front and premolar teeth. It is mostly brought about by chronic inflammation (tartar, excess filling, or crown).

127. Pigmented nevus (intradermalis)

128. Torus Mandibularis

May God be with us and help us nail NBDE PART 2  129. Leukoplakia Simplex (Homogenous)

130. Non-homogenous leukoplakia Verrucous leukoplakia

131.

May God be with us and help us nail NBDE PART 2  132. (Reticular Lichen)

133. Cancer of lower lip

May God be with us and help us nail NBDE PART 2  134. Cancer of floor of mouth

135. Gingival cancer

The disease develops mostly on toothless alveolar ridges, where it spreads from the gingiva to the alveolar mucosa and to the surrounding oral structures. Gingival cancer accounts for 8–12% of oral cavity tumours.

Beside smoking and alcohol, mainly chronic irritating factors (odontolith, prosthesis, etc.) may play a role in triggering gingival cancer. The disease destroys the periosteum and the bone if it reaches them, and loosens the teeth. Gingival cancer is more frequent on the mandibular gingiva than on the maxillary, and mandibular gingival cancer metastasizes regionally more frequently. Metastases occur mainly in the submandibular region. Similarly to oral cavity cancers located elsewhere, it is mainly the disease of the elderly (50–70-year-old) men, and histologically most of the cases are squamous epithelial cancers. Incisional biopsy is necessary.

Differential diagnosis: verruca vulgaris, condyloma acuminatum and verrucous carcinoma. epulis, pyogenic granulomas.

May God be with us and help us nail NBDE PART 2  136. Mucocele

137. Mucous Retention Cyst

138. Periapical Cemento Osseous Dysplasia lower anteriors commonly involved, middle aged women, vitality preserved, Radiolucent to Radiopaque

May God be with us and help us nail NBDE PART 2  139. Actinomycosis Normally found in nose and throat. Symptoms triggering is abscess and surgery Abscess leads to hard red to reddish purple lump , often on the jaw from which comes the condition called lumpy jaw Finally abscess breaks through skin and sinus tract is formed. Find Actinomyces and sulfur granules in the drained fluid.

140. Strawberry Tongue

Scarlet Fever Kawasaki Disease Toxic Shock Syndrome

141. Compound Odontoma

May God be with us and help us nail NBDE PART 2  142. Complex Odontoma

143. Recurrent Apthous Ulcer

144.

It will be mentioned that a white / opaque patch, when stretched disappears or it will be A lesion that blanches, and it’s always “Bilateral”

May God be with us and help us nail NBDE PART 2  145. Petechiae, Purpura, Ecchymoses

Difference in size Petechiae are 1mm -2mm Slightly larger than them but less than 1cm is Purpura Largest is Ecchymoses appx 1 cm or more

May God be with us and help us nail NBDE PART 2  Radiology Cases

1. Abrasion

2. Abrasion

May God be with us and help us nail NBDE PART 2  3. Ameloblastoma

4. Ameloblastoma

5. Ameloblastoma

6. Ameloblastoma

May God be with us and help us nail NBDE PART 2  7. Amelogenesis imperfecta

8. Amelogenesis imperfecta

May God be with us and help us nail NBDE PART 2  9. Ankylosis

May God be with us and help us nail NBDE PART 2  10. Ankylosis

11. Ankylosis

May God be with us and help us nail NBDE PART 2  12. Attrition

13. Basal cell nevus syndrome

May God be with us and help us nail NBDE PART 2  14. Basal cell nevus syndrome

15. Basal cell nevus syndrome

May God be with us and help us nail NBDE PART 2  16. Benign cementoblastoma

17. Benign cementoblastoma

May God be with us and help us nail NBDE PART 2  18. Benign cementoblastoma (residual cyst?)

19. Central giant cell granuloma

May God be with us and help us nail NBDE PART 2  20. Central giant cell granuloma

21. Central giant cell granuloma

May God be with us and help us nail NBDE PART 2  22. Chondrosarcoma

May God be with us and help us nail NBDE PART 2  23. Chronic osteomyelitis with periostitis (Garre's)

24. Chronic osteomyelitis with periostitis (Garre's)

May God be with us and help us nail NBDE PART 2  25. Chronic osteomyelitis with periostitis (Garre's)

26. Chronic osteomyelitis with periostitis (Garre's)

May God be with us and help us nail NBDE PART 2  27. Calcifying epithelial odontogenic tumor

May God be with us and help us nail NBDE PART 2  28. Calcifying epithelial odontogenic tumor

May God be with us and help us nail NBDE PART 2  29. Calcifying epithelial odontogenic tumor

May God be with us and help us nail NBDE PART 2  30. Cleidocranial dysplasia

31. Cleidocranial dysplasia

May God be with us and help us nail NBDE PART 2  32. Cleidocranial dysplasia

33. Complex odontoma

May God be with us and help us nail NBDE PART 2  34. Compound odontoma

May God be with us and help us nail NBDE PART 2  35. Compound odontoma

May God be with us and help us nail NBDE PART 2  36. Compound odontoma

37. Concrescence

May God be with us and help us nail NBDE PART 2  38. Concrescence

39. Concrescence

May God be with us and help us nail NBDE PART 2  40. Condensing osteitis

41. Condensing osteitis

May God be with us and help us nail NBDE PART 2  42. Condensing / sclerosing osteitis

43. Condensing / sclerosing osteitis

May God be with us and help us nail NBDE PART 2  44. Dens evaginatus

May God be with us and help us nail NBDE PART 2  45. Dens invaginatus

May God be with us and help us nail NBDE PART 2  46. Dens invaginatus

May God be with us and help us nail NBDE PART 2  47. Dens invaginatus

48. Dens invaginatus

May God be with us and help us nail NBDE PART 2  49. Dens invaginatus

50. Dentigerous cyst

51. Dentigerous cyst

May God be with us and help us nail NBDE PART 2  52. Dentigerous cyst

53. Dentigerous cyst

54. Dentigerous cyst

55. Dentin dysplasia

May God be with us and help us nail NBDE PART 2  56. Dentin dysplasia

57. Dentin dysplasia

58. Dentin dysplasia

May God be with us and help us nail NBDE PART 2  59. Dentinogenesis imperfecta

60. Dentinogenesis imperfecta

61. Dentinogenesis imperfecta

May God be with us and help us nail NBDE PART 2  62. Dentinogenesis imperfecta

63. Dilaceration

64. Dilaceration

May God be with us and help us nail NBDE PART 2  65. Dilaceration

66. Distodens

67. Distodens

May God be with us and help us nail NBDE PART 2  68.

69. Enamel Pearl

70. Enamel Pearl

71. Enamel Pearl

May God be with us and help us nail NBDE PART 2  72. Enostosis

73. Exostosis

74. Fusion

75. Fusion

May God be with us and help us nail NBDE PART 2  76. Fusion

77. Fusion

78. Gardner's Syndrome

79. Gardner's Syndrome

May God be with us and help us nail NBDE PART 2  80. Gardner's Syndrome

81. Gardner's Syndrome

82. Gardner's Syndrome

83. Gardner's Syndrome

May God be with us and help us nail NBDE PART 2  84. Gemination

85. Gemination

86. Globulomaxillary cyst

May God be with us and help us nail NBDE PART 2  87. Globulomaxillary cyst

88. Globulomaxillary cyst

89. Globulomaxillary cyst

May God be with us and help us nail NBDE PART 2  90. Globulomaxillary cyst

91. Hypercementosis

92. Hypercementosis

May God be with us and help us nail NBDE PART 2  93. Hypercementosis

94. Hypercementosis

95. Hypercementosis

96. Hypercementosis

May God be with us and help us nail NBDE PART 2  97. Hypodontia

98. Hypodontia

99. Hypodontia

May God be with us and help us nail NBDE PART 2  100. Hypodontia

101. Hypodontia

102. Hypodontia

103. Idiopathic osteosclerosis

May God be with us and help us nail NBDE PART 2  104. Idiopathic osteosclerosis

105. Idiopathic osteosclerosis (between premolars)

106. Incisive canal cyst

May God be with us and help us nail NBDE PART 2  107. Incisive canal cyst

108. Incisive canal cyst

109. Keratocyst (OKC)

May God be with us and help us nail NBDE PART 2  110. Keratocyst (OKC)

111. Keratocyst (OKC)

112. Keratocyst (OKC)

May God be with us and help us nail NBDE PART 2  113. Lateral periodontal cyst

114. Lateral periodontal cyst

115. Lateral periodontal cyst

May God be with us and help us nail NBDE PART 2  116.

117. Macrodontia

118. Macrodontia

May God be with us and help us nail NBDE PART 2  119. Macrodontia

120. Macrodontia

121. Macrodontia

May God be with us and help us nail NBDE PART 2  122. Malignant neoplasm

123. Malignant neoplasm

124. Malignant neoplasm

May God be with us and help us nail NBDE PART 2  125. Mesiodens

126. Mesiodens

127. Metastatic neoplasm

May God be with us and help us nail NBDE PART 2  128. Metastatic neoplasm

129. Metastatic neoplasm

130. Metastatic neoplasm

131. Metastatic neoplasm

May God be with us and help us nail NBDE PART 2  132. Metastatic neoplasm

133.

134. Microdontia

May God be with us and help us nail NBDE PART 2  135. Microdontia

136. Microdontia

137. Migration

May God be with us and help us nail NBDE PART 2  138. Migration

139. Migration

140. Migration

May God be with us and help us nail NBDE PART 2  141. Mucus retention cyst

142. Mucus retention cyst

143. Mucus retention cyst

144. Mucus retention cyst

May God be with us and help us nail NBDE PART 2  145. Mucus retention cyst

146. Mucus retention cyst

147. Mucus retention cyst

May God be with us and help us nail NBDE PART 2  148. Multiple myeloma

149. Multiple myeloma

150. Multiple myeloma

May God be with us and help us nail NBDE PART 2  151. Odontodysplasia

152. Odontodysplasia

153. Odontodysplasia

May God be with us and help us nail NBDE PART 2  154. Odontogenic myxoma

155. Odontogenic myxoma

156. Odontogenic myxoma

May God be with us and help us nail NBDE PART 2  157. Osteosarcoma

158. Osteosarcoma

159. Osteosarcoma (mixed radiopacity)

May God be with us and help us nail NBDE PART 2  160. Osteosarcoma (mixed radiopacity)

161. Osteosarcoma (Sunburst)

162. Osteosarcoma (Sunburst)

May God be with us and help us nail NBDE PART 2  163. Osteosarcoma (Sunburst)

164. Osteosarcoma (widened PDL)

165. Osteosarcoma (widened PDL)

May God be with us and help us nail NBDE PART 2  166. Periapical cemental dysplasia

167. Periapical cemental dysplasia

168. Periapical cemental dysplasia

May God be with us and help us nail NBDE PART 2  169. Periapical cemental dysplasia

170. Periapical cemental dysplasia

171. Periapical cemental dysplasia

May God be with us and help us nail NBDE PART 2  172. Periapical cemental dysplasia

173. Periapical granuloma/abcess

174. Periapical granuloma/abcess/radicular cyst

May God be with us and help us nail NBDE PART 2  175. Periapical granuloma/abcess/radicular cyst

176. Periapical granuloma/radicular cyst

177. Periapical Granuloma (widening of apical PDL)

May God be with us and help us nail NBDE PART 2  178. Periapical scar

179. Periapical scar

180. Periapical scar

May God be with us and help us nail NBDE PART 2  181. Periapical scar

182. Periapical scar

183. Primordial cyst

May God be with us and help us nail NBDE PART 2  184. Primordial cyst

185. Primordial cyst

186. Primordial cyst

187. Pulp calcification

May God be with us and help us nail NBDE PART 2  188. Pulp calcification

189. Radicular cyst

190. Residual cyst

May God be with us and help us nail NBDE PART 2  191. Residual cyst

192. Root resorption

193. Root resorption

194. Root resorption

May God be with us and help us nail NBDE PART 2  195. Root resorption

196. Root resorption

197. Root resorption

May God be with us and help us nail NBDE PART 2  198. Root resorption

199. Root resorption

200. Root resorption

May God be with us and help us nail NBDE PART 2  201. Secondary dentin

202. Socket sclerosis

203. Socket sclerosis

204. Stafne's defect

May God be with us and help us nail NBDE PART 2  205. Stafne's defect

206. Stafne's defect

207. Stafne's defect

May God be with us and help us nail NBDE PART 2  208. Suppurative osteomyelitis

209. Suppurative osteomyelitis

210. Suppurative osteomyelitis

May God be with us and help us nail NBDE PART 2  211. Suppurative osteomyelitis

212. Suppurative osteomyelitis

213. Taurodont

May God be with us and help us nail NBDE PART 2  214. Taurodont

215. Taurodont

216. Taurodont

May God be with us and help us nail NBDE PART 2  217. Torus

218. Torus

219. Torus

220. Torus

May God be with us and help us nail NBDE PART 2  221. Transposition

222. Transposition

223. Transposition

May God be with us and help us nail NBDE PART 2  224. Transposition

225. Traumatic cyst

226. Traumatic cyst

May God be with us and help us nail NBDE PART 2  227. Turner's hypoplasia

228. Turner's hypoplasia

Endodontics Diagnosis

Normal Pulp is a clinical diagnostic category in which the pulp is symptom-free and normally responsive to pulp testing. Although the pulp may not be histologically normal, a “clinically” normal pulp results in a mild or transient response to thermal cold testing, lasting no more than one to two seconds after the stimulus is removed. One cannot arrive at a probable diagnosis without comparing the tooth in question with adjacent and contralateral teeth. It is best to test the adjacent teeth and contralateral teeth first so that the patient is familiar with the experience of a normal response to cold.

Reversible Pulpitis is based upon subjective and objective findings indicating that the inflammation should resolve and the pulp return to normal following appropriate management of the etiology. Discomfort is experienced when a stimulus such as cold or sweet is applied and goes away within a couple of seconds following the removal of the stimulus.

Typical etiologies may include exposed dentin (dentinal sensitivity), caries or deep restorations. There are no significant radiographic changes in the periapical region of the suspect tooth and the pain experienced is not spontaneous. Following the management of the etiology (e.g. caries removal plus restoration; covering the exposed dentin), the tooth requires further evaluation to determine whether the “reversible pulpitis” has returned to a normal status. Although dentinal sensitivity per se is not an inflammatory process, all of the symptoms of this entity mimic those of a reversible pulpitis.

May God be with us and help us nail NBDE PART 2  Symptomatic Irreversible Pulpitis is based on subjective and objective findings that the vital inflamed pulp is incapable of healing and that root canal treatment is indicated. Characteristics may include sharp pain upon thermal stimulus, lingering pain (often 30 seconds or longer after stimulus removal), spontaneity (unprovoked pain) and referred pain. Sometimes the pain may be accentuated by postural changes such as lying down or bending over and over-the-counter analgesics are typically ineffective. Common etiologies may include deep caries, extensive restorations, or fractures exposing the pulpal tissues. Teeth with symptomatic irreversible pulpitis may be difficult to diagnose because the inflammation has not yet reached the periapical tissues, thus resulting in no pain or discomfort to percussion. In such cases, dental history and thermal testing are the primary tools for assessing pulpal status. Asymptomatic Irreversible Pulpitis is a clinical diagnosis based on subjective and objective findings indicating that the vital inflamed pulp is incapable of healing and that root canal treatment is indicated. These cases have no clinical symptoms and usually respond normally to thermal testing but may have had trauma or deep caries that would likely result in exposure following removal. Pulp Necrosis is a clinical diagnostic category indicating death of the dental pulp, necessitating root canal treatment. The pulp is non-responsive to pulp testing and is asymptomatic. Pulp necrosis by itself does not cause apical periodontitis (pain to percussion or radiographic evidence of osseous breakdown) unless the canal is infected. Some teeth may be nonresponsive to pulp testing because of calcification, recent history of trauma, or simply the tooth is just not responding. As stated previously, this is why all testing must be of a comparative nature (e.g. patient may not respond to thermal testing on any teeth).

Previously Treated is a clinical diagnostic category indicating that the tooth has been endodontically treated and the canals are obturated with various filling materials other than intracanal medicaments. The tooth typically does not respond to thermal or electric pulp testing.

Previously Initiated Therapy is a clinical diagnostic category indicating that the tooth has been previously treated by partial endodontic therapy such as pulpotomy or pulpectomy. Depending on the level of therapy, the tooth may or may not respond to pulp testing modalities.

Apical Diagnoses Normal Apical Tissues are not sensitive to percussion or palpation testing and radiographically, the lamina dura surrounding the root is intact and the periodontal ligament space is uniform. As with pulp testing, comparative testing for percussion and palpation should always begin with normal teeth as a baseline for the patient. Symptomatic Apical Periodontitis represents inflammation, usually of the apical periodontium, producing clinical symptoms involving a painful response to biting and/or percussion or palpation. This may or may not be accompanied by radiographic changes (i.e. depending upon the stage of the disease, there may be normal width of the periodontal ligament or there may be a periapical radiolucency). Severe pain to percussion and/or palpation is highly indicative of a degenerating pulp and root canal treatment is needed. Asymptomatic Apical Periodontitis is inflammation and destruction of the apical periodontium that is of pulpal origin. It appears as an apical radiolucency and does not present clinical symptoms (no pain on percussion or palpation).

May God be with us and help us nail NBDE PART 2  Chronic Apical Abscess is an inflammatory reaction to pulpal infection and necrosis characterized by gradual onset, little or no discomfort and an intermittent discharge of pus through an associated sinus tract. Radiographically, there are typically signs of osseous destruction such as a radiolucency. To identify the source of a draining sinus tract when present, a guttapercha cone is carefully placed through the stoma or opening until it stops and a radiograph is taken.

Acute Apical Abscess is an inflammatory reaction to pulpal infection and necrosis characterized by rapid onset, spontaneous pain, extreme tenderness of the tooth to pressure, pus formation and swelling of associated tissues. There may be no radiographic signs of destruction and the patient often experiences malaise, fever and lymphadenopathy.

Condensing Osteitis is a diffuse radiopaque lesion representing a localized bony reaction to a low-grade inflammatory stimulus usually seen at the apex of the tooth.

Diagnostic Case Examples

Practice Case 1

Mandibular right first molar had been hypersensitive to cold and sweets over the past few months but the symptoms have subsided. Now there is no response to thermal testing and there is tenderness to biting and pain to percussion. Radiographically, there are diffuse radiopacities around the root apices.

. Diagnosis: Pulp necrosis; symptomatic apical periodontitis with condensing osteitis. Non-surgical endodontic treatment is indicated followed by a build-up and crown. Over time the condensing osteitis should regress partially or totally (15).

May God be with us and help us nail NBDE PART 2  Practice Case 2

Following the placement of a full gold crown on the maxillary right second molar, the patient complained of sensitivity to both hot and cold liquids; now the discomfort is spontaneous. Upon application of Endo-Ice® on this tooth, the patient experienced pain and upon removal of the stimulus, the discomfort lingered for 12 seconds. Responses to both percussion and palpation were normal; radiographically, there was no evidence of osseous changes.

Diagnosis: Symptomatic irreversible pulpitis; normal apical tissues. Non-surgical endodontic treatment is indicated; access is to be repaired with a permanent restoration. Note that the maxillary second premolar has severe distal caries; following evaluation, the tooth was diagnosed with symptomatic irreversible pulpitis (hypersensitive to cold, lingering eight seconds); symptomatic apical periodontitis (pain to percussion)

Practice Case 3

Maxillary left first molar has occlusal-mesial caries and the patient has been complaining of sensitivity to sweets and to cold liquids. There is no discomfort to biting or percussion. The tooth is hyper-responsive to Endo-Ice® with no lingering pain.

May God be with us and help us nail NBDE PART 2  Diagnosis: reversible pulpitis; normal apical tissues. Treatment would be excavation of the caries followed by placement of a permanent restoration. If the pulp is exposed, treatment would be non-surgical endodontic treatment followed by a permanent restoration such as a crown.

Practice Case 4

Mandibular right lateral incisor has an apical radiolucency that was discovered during a routine examination. There was a history of trauma more than 10 years ago and the tooth was slightly discolored. The tooth did not respond to Endo-Ice® or to the EPT; the adjacent teeth responded normally to pulp testing. There was no tenderness to percussion or palpation in the region.

Diagnosis: pulp necrosis; asymptomatic apical periodontitis. Treatment is nonsurgical endodontic treatment followed by bleaching and permanent restoration.

Practice Case 5

Mandibular left first molar demonstrates a relatively large apical radiolucency encompassing both the mesial and distal roots along with furcation involvement. Periodontal probing depths were all within normal limits. The tooth did not respond to thermal (cold) testing and both percussion and palpation elicited normal responses. There was a draining sinus tract on the mid-facial of the attached gingiva which was traced with a gutta-percha cone. There was recurrent caries around the distal margin of the crown

May God be with us and help us nail NBDE PART 2 

Diagnosis: pulp necrosis; chronic apical abscess. Treatment is crown removal, non-surgical endodontic treatment and placement of a new crown

Practice Case 6

Maxillary left first molar was endodontically treated more than 10 years ago. The patient is complaining of pain to biting over the past three months. There appear to be apical radiolucencies around all three roots. The tooth was tender to both percussion and to the Tooth Slooth®.

Diagnosis: previously treated; symptomatic apical periodontitis. Treatment is nonsurgical endodontic retreatment followed by permanent restoration of the access cavity.

May God be with us and help us nail NBDE PART 2  Practice Case 7

Maxillary left lateral incisor exhibits an apical radiolucency. There is no history of pain and the tooth is asymptomatic. There is no response to Endo-Ice® or to the EPT, whereas the adjacent teeth respond normally to both tests. There is no tenderness to percussion or palpation.

Diagnosis: pulp necrosis; asymptomatic apical periodontitis.Treatment is nonsurgical endodontic treatment and placement of a permanent restoration.

May God be with us and help us nail NBDE PART 2  MASTER DAY2 NORMAL RADIOGRAPHIC ANATOMICAL LANDMARKS a : enamel

B : dentin a C : pulp c b Cervical burnout : Radiographs sometimes show Diffuse radiolucent areas With ill defined borders Present on mesial n distal Aspects of teeth in cervical region

These regions appear between The edge of the enamel cap And the crest of the alveolar Ridge.

It should not be confused with Root caries. Lamina dura: It is a thin radio-opaque Layer Of dense bone surrounding the Tooth socket.

It is thicker than the surrounding Trabecular bone and thickness Increases with increase in amount In the occlusal stress Alveolar crest

The gingival margin of the Alveolar process that extends Between the teeth is Apparant on radiographs As radiopaque line called Alveolar crest.

It is considered as normal If it is not > than 1.5 mm From cej of adjacent teeth. Periodontal ligament space PDL space is primarily composed of collagen, so appers as radiolucent Space between tooth root and lamina dura. Cancellous bone : it lies between the cortical plates in both the jaws. It is composed of thin radiopaque plates and rods surronding many Small radiolucent marrow spaces. •Email •Likd × •Save Landmarks of maxilla •Embed

median palatine suture Nasal fossa Nasal septum Anterior nasal spine Incisive foramen Lateral fossa Maxillary sinus Malar bone Maxillary tuberosity Hamular process Nasolacrimal duct Maxillary anterior region

Nasal septum

Nasal fossa

Nasal spine

Insicive foramen

Nose

Median palatine suture e b

a c d

A-Nasal septum D-Incisive foramen

B-Nasal fossa E-Median palatine suture

C-Anterior nasal spine Nasal cavities Nasal septum

Anterior nasal spine

Median palatine suture 1 .Nasal septum (17): appears as a radio-opaque line that separates the two nasal fossa in the midline.

2. Anterior nasal spine (16): appears as a V-shaped radio-opaque structure in the midline above the incisive foramen.

3. Incisive foramen (12): or the anterior palatine fossa, it usually appears as a prominent radiolucent area above or between roots of two central incisors, it usually appears round or oval in shape and doesn’t exceed 6mm in diameter. Palatal view

Median palatine suture Median palatine suture

Appears as a vertically oriented Radiolucent line in true image projections through the midline. Usually prominent between the two central incisor roots in Young individuals. Palatal view

Incisive foramen Red arrow points to periapical cyst periapical lesion Red arrow :mesiodens mesiodens Incisive canal cyst

Facial view

Nasal septum Facial view

Nasal fossa Soft tissues of nose Red arrow : soft tissues of nose Green arrow :lip line Maxillary canine

Floor of nasal fossa

Maxillary sinus

Lateral fossa

Nose a

c

b

a: Floor of nasal fossa b: Maxillary sinus c: Lateral fossa Floor of nasal fossa (red arrow) and anterior border of maxillary sinus (blue arrows),forming the inverted (upside down ) Y Lateral fossa : the radiolucency results from a depression Above and posterior to the lateral incisor. To help rule Out a pathology, look for an intact lamina dura Surrounding the adjacent bone. The maxillary sinus surrounds Black arrows indicate the floor The root of the canine, of the nasal fossa ,the max Which may be misinterpreted sinus (red arrows) has As pathology. pneumatized bet the 2nd PM And 1st molar. Maxillary premolar region

Sinus septum

Zygomatic process

Maxillary sinus d c a c a d

a: Malar process c: Sinus septum d: Maxillary sinus Maxillary molar region

zygoma Maxillary sinus

ptrygoid plate

Hamular process

Coronoid process maxillary tuberosity Maxillary sinus :

The maxillary sinus with Its thin bony walls, its thin mucosa, and its vast Air space, produce an Extremely dark image Deep to the maxillary Teeth.

Its outlines, particularly its Floor, are clearly delineated By delicate radio-opaque lines 1- body of zygoma

6- apices of roots

7 - floor of the sinus

8- septum of the sinus

molar region In general ,the floor of the sinus is Approximately coincidental with The location of the apices of the Roots of the upper teeth (bicuspids And first two molars ).

But there is often as much as two Or even three mm of maxillary Bone between the root ends and Sinus floor. In other cases the sinus floor dips so deeply between the roots of the Maxillary teeth that the latter appear to project into it for as much as One half of their length Pneumanization :expansion of sinus wall into surrounding bone, usually in an area where teeth have been lost prematurely.

It increases with age . e e g g d d f c c a f a b

A :max tuberosity e : zygoma (dotted lines) B :coronoid process f : max sinus C : hamular process g :sinus recess D: pterygoid plates

Image of the imacted third molar superimposed Malar (zygomatic) process. U or j- shaped Radio-opacity, often superimposed over the roots Of the molars, specially when using the bisecting Angle technique. The red arrows define the Lower border of the zygomatic bone. Maxillary tuberosity : the rounded elevation located at the posterior aspect of both sides of the maxilla. Malar bone : or the zygomatic Process. The inferior portion Of the malar bone appears As a radio-opaque, U shaped structure Related to the roots of the first Maxillary molar. It represents the Attachment of the zygomatic bone To the maxilla.

Maxillary tuberosity: appears As a radio-opaque structure that Extends distally and upward from Posterior to maxillary sinus. It Represents the end of maxilla. Black arrows : hamular process Purple arrows : pterygoid plates Coronoid process : it is seen as a radio-opaque structure in maxillay molar iopa. Hamular process: It is a bony spine extending from Pterygoid process of the sphenoid Bone. It appears as a radio-opaque Spine that is recorded on radiographs Distal to the tuberosity of maxilla And extends downward.

Coronoid process of the mandible: Appears as a triangular radio-opaque structure projected into the same general area of max periapical film projections distal to the maxillary teeth. The zygomatic process (green) is a U shaped rationality. Normally the Zygomatic bone posterior to this is very dense and radio-opaque. In this, however the max sinus has expanded into the zygomatic bone and makes the area more radiolucent (red). The coronoid process (orange), the pterygoid plates (blue) and the max tuberosity (pink) are also identified . Nasolacrimal duct : It is seen in occlusal view of the maxilla as a round radiolucent area superimposed on the posterior region of the hard palate. Landmarks of mandible

Lingual foramen Genial tubercles Mental ridge Mental foramen Mental fossa External oblique line Internal oblique line Mylohyoid line Mandibular foramen Inferior alveolar canal Submandibular gland fossa Nutrient canals Mandibular symphysis Symphysis In a new born infant Fracture Mandibular incisor region

Mental ridge mental fossa

Genial tubercle Lingual foramen d c

a b

A: lingual foramen C :mental ridge b: genial tubercles D: mental fossa Lingual foramen : radiolucent ‘hole ’ in centre of genial tubercles. Lingual nutrient vessels pass through this foramen . Lingual foramen

It is set in the midline deep To the apices of the Anterior teeth.

It appears as a small Radiolucent dot at the Symphysis area usually Surrounded by a radio Opaque structure. Genial tubercles : radio-opaque area in the midline surrounding The lingual foramen Genial tubercles:They appear as radio Opaque circle that surrounds the lingual foramen just below the apices of the Of the incisors. mental ridge Mental ridge : it is a bony prominence found on the labial Aspect of the mandible near its inferior border and extended From the premolar region to the symphysis area on wchich It takes an upward turn as it approaches it. It appears as a Radio opaque line below the apices of anterior teeth. mental fossa : this represents the depression on the labial aspect of the mandible overlying the roots of the incisors. The resultant radiolucency may be mistaken for pathology Nutrient canals :

The arrows above identify nutrient Canals. They are most often seen in Persons with thin bone and in Those with high blood pressure Or advanced periodontitis.

Nutrient canals Mental foramen (3)

It appears as a radiolucent, ill defined area between the apices of bicuspids.

it represents the anterior Terminates of the mandibular canal. 6- caries

7- prepared cavity

8- enostosis

9 –mental foramen Mandibular premolar region

A – mylohyoid ridge B - mandibular canal c – submandibular gland fossa d – mental foramen b a d

c

b -Mandibular canal a- Mylohyoid ridge d -Mental foramen c -Submandibular gland fossa mental foramen Mandibular molar region

A – external oblique ridge B – mylohyoid ridge C – mandibular canal D- submandibular gland fossa a

b

c d

A -External oblique ridge B - Mylohyoid ridge c -Mandibular canal D -Submandibular gland fossa External oblique ridge - a continuation of the anterior border of ramus passing downward and forward on the buccal side of the mandible. It apperas as a radio-opaque line which usually ends anteriorly in the Area of the first molar. The red arrow points to the mylohyoid ridge. The external oblique ridge (red) and the mylohyoid ridge (blue) Usually run parallel to each other , with the external oblique ridge Always being higher on the film . Internal oblique ridge -

It appears as a radio opaque Line, descending downwards And forward from coronoid Process, in a horizontal Position, stop at the third Molar area or become cotinuous With the mylohyoid line Its placed below the external Oblique ridge. Mylohyoid ridge (internal oblique) – located on the lingual Surface of the mandible , extending from third molar area to Premolar region Mandibular canal : arises at the mandibular foramen on the Lingual side of the ramus and passes downward and forward, Moving from the lingual side in the third molar region to the buccal Side in the premolar region. It contains the inferior alveolar nerves and vessels. The mandibular canals (red arrows) usually runs very close to the molars

Green arrow showing dilaceration of roots.

The other film shows impaction at the superior border of the canal . Bifid mandibular canal: a) cone beam section b) Cross section Submandibular gland fossa Submandibular gland fossa -

It is a depression on the lingual Aspect of the mandible on which Submandibular glands are Present.

It appears as a zone of Radiolucency below the Lower molars a b

c

d

A – external oblique ridge B – mylohyoid ridge c – mandibular canal D – submandibular gland fossa THANK YOU

DEES DAY 2 CASES ++++

Case 1: old man, has MI, had adenocarcinoma before, took radiotherapy for that, hepatitis before 24b yrs, smokers, knee replacement before 6 months, Medications – aspirin, warfarin, and some other medicines. Oral findings: white patch on floor of mouth, anterior cross bite, missing maxillary molar and exostosis on the buccal of mandibular premolar /molar area, melanotic macule on the palate Questions: 1. Reason for dry mouth- medication 2. Reason for the brown lesion on palate- cigarette use 3. Patch on the floor can be all except: stomatitis nicotinia 4. Which lab test for the viral infection : HgSab ,/ HgcAb/, Transaminase ,/ no test 5. The patient is at increased risk for all except: Procedure related bleeding / Hyposalivation / / SCC 6. Reason for the anterior cross bite : early loss of maxillary molar 7. What is true for his malocclusion Related only to the anterior region ( on the left side he had a missing canine maxillary and the posterior bite looked collapsed) Correction of ant cross bite will improve esthetic and function The premolar will function for the missing canine(smthg like this) 8. Most important to consider when deciding for the antibiotic prophylaxis : time elapsed since the surgery

Case 2: 8 yr old girl with many missing teeth due to caries and poor oral hygiene.. anterior cross bite and a supernumary tooth Questions: 1. All will be included in the ortho informed consent except Ortho treatment can bend roots of the teeth Caries and gums dieses can happen during ortho treatment Injury to the nerve due to any previous accident can be increased(or somthg like that) during ortho During ortho Pt will have to wear mouthguard during sports 2. All are negative sequel of extraction of the supernumary tooth except Necrosis of 7 Non eruption of 7 Necrosis of 8 Necrosis of 6 3. Correction of the anterior cross bite will result in all except Increase maxillary arch perimeter Improve eruption of tooth 11 Stop root development of the central incisor One more option 4. Correction of cross bite ASAP Removable appliance with finger springs is one of the ways of correcting cross bite True/ false question

Case 3: A 20 years old girl , complains of occasional pain in the back lower jaws, has asthma Take albuterol Questions: 1. Reason for her pain 3rd molar Occlusal trauma Caries (in the radiograph and pic there was no clue of 3rd molars or trauma or caries) 2. She starts wheezing on expiration . what will u Not do Steroid inhaler B2 agonist inhaler Put pt is comfortable position Give oxygen

Case 4: 42 yr old man, many teeth missing, wants denture Oral finding : mandibular Canine to canine teeth r present, posteriors all edentulous Maxillary: upper 2nd molar and 2nd premolar present rest all missing Questions: 1. All are reason for his early tooth loss except Poor oral care as a child Lack of fluoridated water Untreated periodontal disesese Incomplete tooth development 2. All are risk of extracƟ on of the upper teeth. Which is LEAST excepted Dry socket Tuberosity fracture damage to adjacent tooth one more option (that was not the least ) 3. For denture in the mandible- least preferred Extracted remaining teeth. Complete denture Extract all anterior except canines on both sides, crown on the canine and removable Extract all teeth, 2 implants and dentures Extract all, implant and fixed from 27- 25 and 22- 24, remaning removable

Case 5 Another case where the patient had edentulous space in the mandibular teeth with 18 present. Has radio opaciƟ es in that edentulous area. Questions: What is least considered when planning for an implant. Extraction of 18 Excision of the radioopacity Proximity to vital structures One more- don’t remeber( it was important consideration) Random true/false that remember 1. Smoking is a contributing factor for Accumulated calculus causes periodontal disease 2. Extraction of 32 causes damage to the lingual nerve Afer extraction of 32 the soft tissue should be biopsied 3. Periodontal disease will inevitably cause tooth mortality And one more regarding tooth mortality. Don’t remember

Case 1: old man , has MI, had adenocarcinoma before, took radiotherapy for that, hepatitis before 24b yrs, smokers, knee replacement before 6 months, Medications – aspirin, warfarin, and some other medicines Oral findings: white patch on floor of mouth, anterior cross bite, missing maxillary molar and exostosis on the buccal of mandibular premolar /molar area, melanotic macule on the palate Questions: 1. Reason for dry mouth- medication 2. Reason for the brown lesion on palate- cigarette use 3. Patch on the floor can be all except: stomatitis nicotinica 4. Which lab test for the viral infection : HgSab ,/ HgcAb/, Transaminase ,/ no test 5. The patient is at increased risk for all except: Procedure related bleeding / Hyposalivation / Osteoradionecrosis / SCC 6. Reason for the anterior cross bite : early loss of maxillary molar 7. What is true for his malocclusion Related only to the anterior region ( on the left side he had a missing canine maxillary and the posterior bite looked collapsed) Correction of ant cross bite will improve esthetic and function The premolar will function for the missing canine(smthg like this) 8. Most important to consider when deciding for the anƟbioƟc prophylaxis : Ɵme elapsed since the surgery

Case 2: 8 yr old girl with many missing teeth due to caries and poor oral hygiene.. anterior cross bite and a supernumary tooth Questions: 1. All will be included in the ortho informed consent except Ortho treatment can bend roots of the teeth Caries and gums dieses can happen during ortho treatment Injury to the nerve due to any previous accident can be increased(or somthg like that) during ortho During ortho Pt will have to wear mouthguard during sports 2. All are negative sequel of extraction of the supernumary tooth except Necrosis of 7 Non eruption of 7 Necrosis of 8 Necrosis of 6 3. Correction of the anterior cross bite will result in all except Increase maxillary arch perimeter Improve eruption of tooth 11 Stop root development of the central incisor One more option 4. Correction of cross bite ASAP Removable appliance with finger springs is one of the ways of correcting cross bite True/ false question

Case 3: A 20 years old girl , complains of occasional pain in the back lower jaws, has asthma Take albuterol Questions: 1. Reason for her pain 3rd molar Occlusal trauma Caries (in the radiograph and pic there was no clue of 3rd molars or trauma or caries) 2. She starts wheezing on expiration . what will u Not do Steroid inhaler B2 agonist inhaler Put pt is comfortable position Give oxygen

Case 4: 42 yr old man, many teeth missing, wants denture Oral finding: mandibular Canine to canine teeth r present, posteriors all edentulous Maxillary: upper 2nd molar and 2nd premolar present rest all missing Questions: 1. All are reason for his early tooth loss except Poor oral care as a child Lack of fluoridated water Untreated periodontal disesese Incomplete tooth development 2. All are risk of extraction of the upper teeth. Which is LEAST excepted Dry socket Tuberosity fracture damage to adjacent tooth one more option (that was not the least ) 3. For denture in the mandible- least preferred Extracted remaining teeth. Complete denture Extract all anterior except canines on both sides, crown on the canine and removable Extract all teeth, 2 implants and dentures Extract all, implant and fixed from 27- 25 and 22- 24, remaning removable

Case 5 Another case where the patient had edentulous space in the mandibular teeth with 18 present. Has radio opacities in that edentulous area. Quest was smthing like What is least considered when planning for an implant. Extraction of 18 Excision of the radioopacity Proximity to vital structures One more- don’t remeber( it was important consideration) Random true/false that remember 1. Smoking is a contribuƟng factor for periodontal disease Accumulated calculus causes periodontal disease 2. Extraction of 32 causes damage to the lingual nerve AƟer extraction of 32 the soft tissue should be biopsied 3. Periodontal disease will inevitably cause tooth mortality And one more regarding tooth mortality. Don’t remember

CASES 1. 8 yearold with poor oral hygiene.lives with mom and brother.many extractions at age 5..notallergic to any jus topical meds for any allergic reactions if occur. She has anterior cross bite in pics..lat to lat max.. what happens to facial gingiva ? What treatment to correct ? Why did it occur ? How do you do that in detail ? What you tel for good oral hygiene maintenance- it will affect your ortho tx/ give oh instructions and follow up/ electric tooth brush ? 2. 75yr old male, smokes sigarette since Syears. Has all medical condnts.. M|, Knee replacement,diabetic and so on.. many medications taken..what causes dry mouth? Has lost back many back teeth and want dentures...Why did he loose teeth ? Why is his oh bad ? Perio quesns on that ? Do you make cd or rpd with this conditions? What medical precautions u take before any surgery 7 Has a filling mo on 32 and if u make rpd what do you do for 32 ? Biz he has do caries which is not treated..amalgam compaosite or crown ? 3. 45 year old lady,lives with son. Many teeth lost and want dentures to chew food. Lat to lat max trauma 30years ago. They are treated and now they look perfect. All rct done..they ask a quesn on max central what's the problem? But it's a perfect rct and no need to redo .. quesn on guttapercha of those trauma treated teeth.. how they look ? Canine broken crown which was rct done previously. Quesn was they do ortho to extrude a lil bit so that what advantage happens 7 No need for crown lengthening/ sufficient for crown/ can we make fpd ?

Case 1- Patient with prosthetic knee replacement 6 month ago, H/O bypass surgery and radiotherapy for prostate adenocarcinoma. Taking many medications. Q 1 Reason for dry mouth? Xerostomia due to medication Q 2 He need antibiotic prophylaxis for what reason? Recent knee replacement Q 3 Has a white lesion on floor of mouth..It could be anything except SCC. Verrucous carcinoma, Nicotinic stomatitis? Nicotinic stomatitis

Case 2- Girl 8 years..multiple extracted teeth due to caries, anterior cross bite, supernumerary tooth Q 1 When should be treated for cross bite? During erupting Q 2 Preventive approach for caries? Pit and fissure sealants Q 3 Maxillary 1st molar tilted (due to early loss of E)..what should be the treatment? Space regaining appliance / Band and loop

Case 3: little girl 9 1/2 yrs old (w/ unerupted canine and pm).this child has allergy and taking cetirisine., Q1. wat side effect this drug has?: internal staining,ext staining, dry mouth?Dry mouth Q2. teeth age of this girl which still have not erupted canine? which is older than 8,younger than 8, Below 9 years...? Below 9 years Q3. this girl had lots of caries too on occlusal and Mes and distal of posterior teeth. which tx: amalgam,composite, ss crown? ss crown (if caries involves more than 1 surface, go with SSC acc to DD)

Case 4: woman pat 48 yrs old w bells palsy which happened a month ago: Q1. effect of bells palsy how it recovered? Q2. wat do you say to pt.? options: it will healed , self limited, healed by partially problem, permanent problem.? self limited

Case 5: 68yrs old man w/ lots of meds and condition,angina,bis phosphonate.. he needs extraction. Q1. about prophylaxis? Q2. about time of extraction?(relating to bisphospho? hyperbaric oxygen Therapy 3 months to extraction

Case 6: case on asthmatic pt, picture of condyles on panoramic and Q1. pt have osteoarthritis or rheumatoid arthritis? Rheumatoid (its asthma related)

Case 7: ANB angle is 5.8 and he in the clinical picture have cross bite anterior what will be skeletal class? Class II

Case 8: Pedo, one was routine examination but found caries and what is tx for each tooth, about his behavior and patient management, space maintenance, number of permanent teeth seen on pano. She had a shunt placed some years ago. His pano had a oval radiolucency near the condyle on both the sides Q1. for what it was? all anatomical landmarks like external meatus or transverse canal etc?

Case 9: Pedo, was a girl with class 3 in primary teeth, although intraoral pics didn’t show primary 2Ms, but anterior were edge to edge. She lost a lot of space Q1. the cause of space loss, and space management, not space maintenance (look for small words in the questions to answer wisely) Q2. her facial profile? Q3. her oral hygiene practices were poor, how to motivate her? Voice control, negative or positive reinforcement? positive reinforcement

Case 10: Adult, a man with mand tori identification on pano, with no significant med history but takes bisphosphonates. Q1. how would you modify your plans? Case 11: Adult, lady, had ortho done when she was teenager, now has upper front teeth lost, she is about 40’s now. Q1. reason for spaces Q2. she had chelitis angularis, reason to that? Fungal/ vit B def Q3. she had facia palsy, what would you tell the pt about the prognosis of this long term disease? Q3. prostho management, placement of clasps, materials to be used, some teeth look like their restorations are old, what will you treat these teeth with? (don’t get if her radiographs and no. of teeth seen on x rays don’t match her clinical teeth)

Case 12: Adult, 50 yr up pt, she had trauma some time ago and lower 3 teeth were discplored, upper right CI was RCt, apicectomy treated and she also had tori, but almost all teeth present. Q1. what will you do about the tori? Q2. what about the fractured crown, redo or repair? Q3. the upper CI periapical lesion did not heal in 2 years what can it be? And how to treat it? Q4. RCT bleaching and crowns ?

Case 13: Adult, young lady with regular dental tx, on OCP Q1. what meds not to prescribe? Carbamazepine (Carbatrol, Epitol, Equetro, Tegretol), Felbamate (Felbatol), Oxcarbazepine (Trileptal), Phenobarbital (Luminal), Phenytoin (Dilantin, Phenytek), Primidone (Mysoline), Topiramate (Topamax) Q2. she had a palatal lesion, differentials? Q3. she had unknown swellings in mand right post, vital teeth, differentials? Q4. extracted the third molar but cant resolve the lesion? OKC (coz microscopy said they found epithelial cells and inflammatory cells) Q5. pdl management phases? Q6. Hep a treated previously, what should u keep in mind? it is not a blood borne disease Q7. if any special care or precautions needed?

Case 14: Adult, 90 year old man comes with his son as guardian, he has had tube ligation done, some anti hypertensive tx, several teeth missing, mand psot ridge knife edge, he thinks his dentures doesn’t fit any more. Q1. his prostho tx? Q2. had one radiopacity between two teeth, what can be dx? idiopathic osteosclerosis

Case 15: Adult, pt with very very poor OH, and retained root pieces, 3M present but no first or second molars in some places, lower both 3M were semi-impacted and mesioangulated, he had ameloblastoma, he had this drug for depression, for allergies, etc, and his treatment was based on early, and late treatment plans, kinds of prostho tx, clasps, crowns, materials of choice, etc

Case 16: Restorations Q1. height copper and low copper restoration? check X-ray and it has to be same, So look at the photos and choose the different color. high copper content of amalgam are superior to conventional amalgam because less likely to corrode and have less marginal breakdown, main purpose of adding copper to amalgam is to decrease gamma 2 phase(tin mercury)

Case 17: ortho patient Q1. has canine class 1 but ob and oj is minimal? mand crowding cuz tooth size are normal Q2. Dental and chronical age Q3. about RPI?

Case 18: Guy from Ethiopia had Hep B positive surface antibody- Q1. what does this mean? he is fine, he has it and needs vaccine, he has it and needs some medication? He is fine (previous exposure with hep B / must be vaccinated and he is not a carrier)

Case 19: a scenario with a young girl Q1. who had a trauma tooth with a “bubble on the gums”, she also drank a ton of soda? Bubble on gum is related to absess

Case 20: some man who was very fat, had HT, - Q1. what else can he have? Diabetes Q2. which organ would be least effected knowing his condition… kidney, pancreas, thyroid or colon-? Colon Q3. what was least likely to cause his high caries rate? dry mouth (could be cariogenic food, because in the case didn't say anything about what he is eating or if he drinks a lot of water) Q4. Thyroid ? Htn never affects Thyroid whereas a thyroid disorder causes Htn

Case 21: A man who is smoker with knee replacement 6 month ago, White lesion under the tongue. Brown pigment near midline of palate. Hypertensive, took many meds and he had weird occlusion of post class II and anterior cross bite. Missing upper canine (premolar took place and function as a canine, noticed it was missing when asked history) and the other side, he was missing 1M. Q1. The decision to give prophylactic antibiotic is based on: According to the patient AND physician recommendation or preference? Location of knee surgery Extend of knee surgery The years elapsed from the surgery Q2. The cause for this pt occlusion is: Class II molar and canine relationship Class III molar and Canine relationship Early loss of a maxillary tooth Q3. What to do with the lesion, cytology or bio psy? Biopsy Q4. if you would do emergency surgery to this pt, the MOST you would concern is Prolonged bleeding or Cardiovascular issues? CVS (because of profound bleeding not prolonged, he was not on aspirin) Q5. Lesion could be any of these except: SSC, verrucous carcinoma, keratosis, actinic stomatitis? Nicotine Stomatitis (reverse smoking to be exact cause smokers keratosis.. somekeless wont appear on palate.. they cause Verrucous Carcinoma. Nicotine melanosis and stomatitis both by smoking) Q6. The pigment is most likely caused by? smoking Q7. The biopsy of the lesion came back with increase melanin in epithelial layer so the lesion is? nicotine stomatitis-increased production of melanin by the normal number of melanocytes(melanin 1q) A benign lesion on the hard palate typically associated with pipe and cigar smoking.(smoke 2q) Q8. RO body in upper 1PM socket? remaining root tip

Case 22: the case for young child, chephalometry, SNA and SNB no are given, and diagnosis of skeletal class. Know them.

Case 23: A case about an 8 yr girl, early loss of upper M1 and M2, canine tilted distaly and perm 1M tipped mesial making the occlusion Class II posterior and cross bite anterior. Impacted upper lateral with superneumary tooth blocking it. Q1. effect of Orthodontic movement Q2. When to correct ANT cross-bite: as soon as possible or wait till complete root formation of upper incisors. As soon as possible Q3. Will ant cross-bite cause movement of lower incisors? Yes Q4. Gingival recession in lower incisors? True

Case 24: A case about a 22 yr old female with bad OH, Q1. Pt needed to extract 2 lower impacted 3rd molars, do we do soft tissue biopsy BEFORE extraction? No (done after extraciton) Q2. Extraction of lower third molar has risk of? lingual nerve ingury Q3. When you do an endo for lower molar with bad perio condition and a cusp breaks, what to do? Place a full ceramic crown Place a ceramo-metallic crown Place a temporary crown until the perio is ttted Q4. How to motivate the pt for proper OH you do all except Positive reinforcing Pt something Operanetconditioning Motivational intervirwing Carrot stick

Case 25: Diabetes case Q1. Diabetes pt what is preferred -FPD or RPD? RPD Q2. Palate with swelling and histological feature like peudostatifiedcolumner epithelium and no effect on underlining cortical bone what is that pleomorphic or exostosis? pleomorphic Q3. Case of HTN take medication lisinopril what happen? orthostatic hypotension Q4. Aspirin 81mg - needs to stop during extraction if root tip and its easy to extract ? No need to stop Q5. Pt taking HTn drug, aspiring, amytriplin for what u need to consult physician? For asprine , consult with physician, Avoid NSAIDS with ACE, Beta Q6. Good canine, restored lateral and need rpd for pm and molars on canine one side and molars on lateral side, What do u not need? no need to correct canine bcz its not carious

Case 26: of 5 lower incisors and all have PFM and not good perio health. What you have to prescribe? Mouthwash without alcohol , flossing daily ? Or another mothwasht? Q1. hypo-salivation.? Mouthwash without alcohol Q2. Amytryplin cause? hyposalivation

Case 27: 11 years old, kidney dialysis for 10 years and got transplant 1 year ago. He had Hodgkin lymphoma 5 years ago, mitral valve and regurgitation. He is taking lot of complex medicines RG and clinical pictures shows that he has amelogenisis imperfecta Q1. all are immunocompromised drugs except? know all immunocompromised names and corticosteroids : Glucocorticoid , hydrocortisone, methylprednisolone, prednisone , ( triamcinolone , beclomethasone, budesonide, flunisolide) these are inhaled corticosetetiod for astham treat. Other immunosuppresive, cyclosporin, azathioprine, methotrexate, cyclophosamide Q2. what drug can cause amelogenisis imperfecta? tetracycline cause amelogensis imperfecta Q3. why his third molars are missing? Third molars do not erupt by 11 yrs age Q4. bilateral radioopacity in mandible whats the dx? Cherubism Q5. in a Rg canine was short in length whats the dx? AI, DI, Dentin dysplasia? DD Q6. does he need Ab before procedures? No According to new guide line mitral valve or without regurgrition dont need AB , check this in dentin Q7. why he has gingival enlargement? He was taking cyclosporine too

Case 28.: 14 years old, all 4 canines erupted buccally and has pigmented macules on her cheek, asthmatic taking albuterol Q1. albuterol can cause all except? increased salivary secretion Q2. small white lesions on palate? Cause of inhaler its candidiasis Q3. is nitrous oxide is contraindicated? Not contraindicated for Asthma Q4. Will you explain the whole ortho tx to her parents and post complications like she may need gingival grafts? Yes Q5. The reason of pigmentation on her cheek? Proliferation of melanocites, proli of basement cells, deposition of melanin or foreign body? Deposition of Melanin (According to DD) Q6. will ectopically canine resorb #7 roots? True Q7. Anb 6, class 1, 2, 3? Class 2 Q8. Clinicall picture what class, it was? class 1 Q9. Features of her face has everything except? incompetent lips Q10. If she decided to extract premolars what forceps not to used? 150 upper 151 lower Q11. In this case there the best treatment can be? a) extract all canines b) expansion of upper and lower arch? expansion of upper and lower arch Q12. the profile? convex depending in the photograph Q13. what is the preventive treatment for this patient? a) sealant for #3,14, 19, 30 b) encourage the patient to use brushing and dental floss c) use mouth wash? sealant for #3,14, 19, 30 Q14. what is the treatment of choice for tooth #19 if it is already catch the prob during examination? a) compsite filling b) sealant c) no treatment? Composite filling (sealent is best for age 6-12 according to DD) Q15. What is the most costly to do to prevent declassification around the braces? a) varnish every 6 month b) scaling every 3 month c) daily systemic supplement d) using mouthwash? Varnish every 6 months

Case 29:, 45 years male, 2 pack smoke a day, dry mouth, lot of carious teeth, went successful rehab for bad alcohol habits, seems he doesn’t drink now Q1. will you prescribe Acetaminophen/oxycodone in this patient? no Q2. missing canine will make max rpd compromised? True Q3. If you use #7 in rpd will it compromise the tooth? Yes cause no posterior teeth and no canine Q4. Rg picture shows tori in maxilla and mandible both Q5. 2*3 radioopacity on LI which has RCT on it, what is it? It is hypercementosis and will you biopsy it Q6. photo showing the patient has preparation about 0.5 from facial and incisal, what type of restoration the patient lost? a) crown b) Veneer c) composite? Veneer Q7. why not prescribe acetaminophen/ oxycodone on this pt?? pt. is alcoholic and cause hepatotoxicity

Case 30: middle age female, smokes daily and she is fed up from falling restorations every time and she wants to extract her all teeth, psoriasis in hands and feet Q1. by doing what patient want, is conflict bw what two, autonomy, justice, nonm, bene? Autonomy and nonmal Q2. treatment options for her? Crowns Q3. what clasp will you give in max RPD if you class II kennedy? RPI Q4. why you can see condyles in PAN, bilateral fracture, osteoarthritis, rheumatoid arthritis? Rheumetoid arthritis Q5. radiolunceny in bw 8 and 9 it was? incisive foramen Q6. if you want her to quit smoking the day of extraction would be the quit date and you give Chantix 1 week beore the quit date? True Q7. consent

Case 31: An Old woman with Parkinson Disease came to the clinic with her hus- band. She had distal caries on maxillary molar. Q1. Out of all the symptoms of Parkinson's disease which symptom is not important to dental treatment? A. Rapid Eye blinking B. Tremor C. Muscle rigidity D. Loss of Automatic Movements Q2. While working, the patient moved, and the dentist injured her near the cheek. Which artery was injured and caused bleeding? A. Buccal artery of maxillary artery B. Labial artery of ECA C. Facial artery of ECA Q3. What is the first important thing to do after the patient starts to bleed? A. Stop bleeding B. Call an oral surgeon C. Call her husband. D. Inform patient

Case 32: A 32 years old lady with cervical neoplasia comes to your clinic for ulcers on one side of her palate. Drug history of taking oral contraceptives. Q1. Action of Oral contraceptives? A. Dec. Lh B. Inc Lh C. Inc Fsh D. Dec. Fsh Q2.What could be the cause of the ulcer A. CMV B. EBV C. HPV Q3. Patient is most likely to have which neoplasia A. HIV B. Cervical cancer (can also be true, depends on details) C. Rubeola Q4. What can be done for diagnosis of this viral disease except? A. Saliva examination B. Examination of fluid from vesicles C. Oral examination (can also be true, depends on details) Q5. Epithelium of this ulcer A. Orthokeratinised B. Parakeratinised C. Nonkeratinised

Case 33: Pedo, routine examination but found caries and what can be tx for each tooth, his behavior and patient management, space maintenance, number of permanent teeth seen on pano.? She had a shunt placed some years ago His pano had a oval radiolucency near the condyle on both the sides, what it was? external auditory meatus

Case 34: Pedo, a girl with class 3 in primary teeth, intraoral pics didn’t show primary 2Ms, but anterior were edge to edge (look for stuff like that to answer such questions). She lost a lot of space, the cause of space loss and space management? (not space maintenance, so look for small words in the questions to answer wisely) her facial profile, her oral hygiene practices were poor, how to motivate her? Voice control, negative or positive reinforcement? Positive reinforcement

Case 35: Adult, a man with mand tori identification on pano, with no significant med history but takes bisphosphonates, how would you modify your plans? Take a note that he takes bisphosphonates, so answers will go accordingly

Case 36: Adult, lady, had ortho done when she was teenager, now has upper front teeth lost, she is about 40’s now, reason for spaces, she had chelitis angularis, reason to that, and she had facia palsy, what would you tell the pt about the prognosis of this long term disease? Simple prostho management, placement of clasps, materials to be used, some teeth look like their restorations are old, what will you treat these teeth with? don’t get confused if clinical and radiographic count of teeth do not match. Sometimes questions from that quadrant having doubtful count might not come.

Case 37: Adult, 50 up pt, she had trauma some time ago and lower 3 teeth were discolored, upper right CI was RCT, apicectomy treated and she also had tori, but almost all teeth present, what will you do about the tori? Q1. what about the fractured crown? redo or repair Q2. the upper CI periapical lesion did not heal in 2 years what can it be? And how to treat it? Q3. simple RCT bleaching and crowns?

Case 38: Adult, young lady with regular dental tx, on OCP, what meds not to prescribe, and she had a palatal lesion, differentials? she had unknown swellings in mand right post, vital teeth, differentials? Q1. extracted the third molar but cant resolve the lesion, was? OKC, coz microscopy said they found epithelial cells and inflammatory cells Q2. pdl management phases? Q3. she had Hep A treated previously, what should u keep in mind? it is not a blood borne disease

Case 39: Adult, 90 something year old man comes with his son as guardian, he has had tube ligation done, some anti hypertensive tx, several teeth missing, mand psot ridge knife edge, he thinks his dentures doesn’t fit any more, t his prostho tx? Q1. he had this one radiopacity between two teeth, dx? idiopathic osteosclerosis

Case 40: Adult, pt with very very poot OH, and retained root pieces, 3M present but no first or second molars in some places, lower both 3M were semi-impacted and mesioangulated, he had ameloblastoma, he had this drug for depression, for allergies, etc, and his treatment was based on early, and late treatment plans, kinds of prostho tx, clasps, crowns, materials of choice?

Case 41: A lady of 50s..black pigment on palate. Q1. which black pigment is not present in oral cavity? Lentigo (LENTIGO MALIGNA à usually occurs in the elderly. It is most common in sun damaged skin on the face, neck, and arms (Hutchinson freckle). Q2. Pt has some non- painful, hard, movable swelling in the floor of the mouth (pic shown) pt is not aware about it? Lymphoepithelial cyst Q3. Pt has some non-painful lesion in 2nd molar region. Pt is not aware of the lesion. which of the following can be D/D but NOT? Lateral periodontal cyst Q4.O Keratocyst - Multiple lesions found in children may be a component of the? nevoid basal cell carcinoma syndrome (gorlin syndrome). the chief site of involvement is the mandible in the posterior body and ascending ramus. often associated with impacted tooth. tendency to grow in an anterior-posterior direction without bony expansion

Case 42: A child with missing lower right 2nd primary molar... Q1. Space loss is due to? mesial & distal drifting of both ant & post teeth Q2. what kind of occlusion? class 1 on left class 3 on right Q3. Where does the chronic abscess seen in primary teeth? Furcation Q4. How to maintain the space for the missing 2nd primary molar with drifting of two adjacent teeth? we cannot as space is lost, we need space regainer its an ASDA ques

Case 43: child case Q1. pt has multiple class 1 caries what filling? Amalgam as amalgam is preferred by the boards other reason composite has C factor and GiC has less strength

Case 44: elderly male 40 pack year cigarette smoking history with multiple drugs, gastric bypass, hypertension? Q1. What can change this pt to stop the habbit? Self motivation through behavior education Q2. Behaviour of the society can be modified by? a) Surveying b) Study conduction? conduction

Case 45: pt with upper and lower few teeth. Pt has financial restriction Q1. How to raise his occlusion? by fabrication of upper complete denture

Case 46: Case on Management of transient ischemic attack- read the drugs? – antiplatelet agents are recommended over anticoagulants to reduce risk. Combining aspirin with dipyridamole is suggested over aspirin alone. Clopidogrel is a reasonable substitute for people allergic to aspirin. A transient ischemic attack (TIA) is a brief episode of neurologic dysfunction caused by ischemia (loss of blood flow) – either focal brain, spinal cord, or retinal – without infarction (tissue death). TIAs have the same underlying cause as strokes: a disruption of cerebral blood flow (CBF). Symptoms caused by a TIA resolve in 24 hours or less. Antiplatelet medications such as aspirin are generally recommended. They reduce the overall risk of recurrence by 13% with greater benefit early on. The initial treatment is aspirin, second-line is clopidogrel (Plavix), third-line is ticlopidine. If TIAs recur after aspirin treatment, the combination of aspirin and dipyridamole may be recommended. Some people may also be given modifiedrelease dipyridamole or clopidogrel. An electrocardiogram (ECG) may show atrial fibrillation, a common cause of TIAs, or other abnormal heart rhythms that may cause embolization to the brain. An echocardiogram is useful in detecting a blood clot within the heart chambers. Such people may benefit from anticoagulation medications such as heparin and warfarin.

Case 47: 11 years old, kidney dialysis for 10 years and got transplant 1 year ago. He had Hodgkin lymphoma 5 years ago, mitral valve and regurgitation. He is taking lot of complex medicines. RG and clinical pictures show he has? amelogenisis imperfect Q1. All are immunocompromised drugs except? know all immunocompromised names and corticosteroids: Glucocorticoid, hydrocortisone, methylprednisolone, prednisone, (triamcinolone, beclomethasone, budesonide, flunisolide) these are inhaled corticosetetiod for astham treat. Other immunosuppresive, cyclosporin, azathioprine, methotrexate, cyclophosamide Q2. What drug can cause amelogenisis imperfecta? Tetracycline Q3. why his third molars are missing? he is 11 year still third molar not erupted Q4. Bilateral radioopacity in mandible whats the dx? Q5. in a Rg canine was short in length whats the dx? AI, DI, DD? Dentin dysplasia Q6. does he need Ab before procedures? no need to antibiotic Q7. why he has gingival enlargement? He was taking cyclosporine too, cyclisporine lead to gingival enlargement

Case 48: 14 years old, all 4 canines erupted buccally and has pigmented macules on her cheek, asthmatic taking albuterol Q1. albuterol can cause all except? increased salivary secretion Q2. small white lesions on palate? Cause of inhaler it is? candidiasis Q3. is nitrous oxide is contraindicated? NO

Q4. Will you explain the whole ortho tx to her parents and post complications like she may need gingival grafts? YES Q5. The reason of pigmentation on her cheek? Deposition of melanin Q6. will ectopically canine resorb #7 roots? YES Q7. Anb 6, class 1, 2, 3? 2 Q8. Clinical picture what class? class 1 Q9. Features of her face has everything except? incompetent lips, her lips looked fine

Case 49: 45 years male, 2 pack smoke a day, dry mouth, lot of carious teeth, went successful rehab for bad alcohol habits, seems he doesn’t drink now Q1. will you prescribe Acetaminophen/oxycodone in this patient? no Q2. missing canine will make max rpd compromised? YES Q3. If you use #7 in rpd will it compromise the tooth? Yes cause no posterior teeth and no canine Q4. Rg picture shows tori in maxilla and mandible both Q5. 2*3 radioopacity on LI which has RCT on it, it is? hypercementosis and will you biopsy it

Case 50: middle age female, smokes daily and she is fed up from falling restorations every time and she wants to extract her all teeth, psoriasis in hands and feet Q1. by doing what patient want, is conflict bw what two, autonomy, justice, nonm, bene? autonomy, nonmalficiency Q2. treatment options for her? Q3. what clasp will you give in max RPD if you class II kennedy? RPI Q4. why you can see condyles in PAN? Rheumatoid arthritis Q5. radiolunceny in bw 8 and 9 it is? incisive foramen Q6. if you want her to quit smoking the day of extraction would be the quit date and you give Chantix 1 week beore the quit date? True Q7. consent

Case 51: An Old woman with Parkinson Disease came to the clinic with her hus-band. She had distal caries on maxillary molar. Q1. Out of all the symptoms of Parkinson's disease which symptom is not important to dental treatment? Rapid Eye blinking Q2. While working, the patient moved, and the dentist injured her near the cheek. Which artery was injured and caused bleeding? Buccal artery of maxillary artery Q3. What is the first important thing to do after the patient starts to bleed? Stop bleeding

Case 52: A 32 years old lady with cervical neoplasia comes to your clinic for ulcers on one side of her palate. Drug history of taking oral contraceptives. Q1. Action of Oral contraceptives? Oral Contraceptives: Ovulation is inhibited by suppression of FSH and LH.? Dec. Lh and Dec. Fsh

Q2.What could be the cause of the ulcer? HPV Q3. Patient is most likely to have which neoplasia? Cervical cancer Q4. What can be done for diagnosis of this viral disease except? Oral examination

Case 53: Cases with asthma women. She takes albuterol. Q1. What can she has? xerostomia. Q2. She had an attack what can you give her, choose three? b.oxygen c.steroids d. beta2 agonist Q3. Reason of asthma? constriction of brionchole. And inflammation of brionchole. Read Q4. What happens in asthmatic bronchiole? constrict

Case 54: some man who was very fat, had HT, - Q1. what else can he have? Diabetes (can also be Thyroid coz Htn never affects Thyroid whereas a thyroid disorder causes Htn) Q2. which organ would be least effected knowing his condition? kidney, pancreas, thyroid or colon? Colon Q3. what was least likely to cause his high caries rate? dry mouth(could be cariogenic food)

Case 55: F/28 years old/ healthy just taking antihistamines…everything else was fine. Q1. Crown on #4 with recession…u can see a little metal on the gingival. She wants to cover it what to do? A whole new crown Q2. Amalgam on the buccal groove of tooth #19, she wants that “black dot” off because of esthetics, what to do? Composite Q3. Missing tooth #20, if she gets a coil spring on #19 how will the moment work…will it push the molar distal only, push the PM mesial…make force on BOTH tooth? Q4. A nevus on her cheeks on the external examination pics. Is it Melanin proliferation, melanin deposition?

Case 56: case of the 10 y/o who had kidney transplant and hodkins. Taking Cellcept, prednisone, cyclosporin Q1. Which drug of the long list he had is immunosuppressive? mycophenolic acid- CellCept Q2. Why does he have amelogenesis? genetic Q3. Why does he have gingival enlargement? Cyclosporine Q4. On his xray, he had a vertical radiolucent line, bilateral on his molars. In the middle of the body of the mandible what was it? Q5. why doesn't he have 3rd molars? genetics (if age is not in option)

Random case and pic questions

1. Picture: “erythematous, bleeding swelling” mandibular swelling right next to premolars on R side? pyogenic granuloma

2. Picture of vermillion border damage on lower lip. And lesion >10cm require? incisional biopsy 3. opg and clinical picture, identify gemination or fusion. gemination there is single pulp canal 4. pano of zxygomatic process of the maxilla .same xray found in dd 5. Radiograph with multiple periapical radiolucencies.( cementosseous dysplasia)? Florid cementoosseous dysplasia 6. Radiograph of cementoblastoma, stafne cyst? stafne cyst is by lingual, angle of mand, and it is radioluc, and cementobl is radioopaque, at apical of molars, you can see it at the mandible angle, but it is tilted to lingual if the angle 7. Panoramic. all mandibular molar with amalgam filling. Which one is low copper?? Deep filling 8. photo showing the patient has preparation about 0.5 from facial and incisal, what type of restoration the patient lost? Veneer 9. Picture said: “erythematous, bleeding swelling” mandibular swelling right next to premolars on R side? pyogenic granuloma 10. Pic like multiple pyogenic granuloma interdental ? ewing sarcoma 11. Lymph node pic in floor of mouth? lymphoepithelial cyst 12. Pic history of new ceramic crown 7 days back two red spots adjacent caused by – herpes virus, apthus ulcer, chemical burn? apthus ulcer 13. Pic of lesion near lip- recurrent herpes 1, recurrent herpes 2, recurrent apthus, candidiasis? recurrent apthus 14. Pic of Lesion under denture - irritation fibroma, papilloma, periapical granuloma? irritation fibroma 15. MUBL? for retrusion 16. DUML? for protrusion 17. dis advantage of widman flap and the procedure of widman flap? Does not eliminate pocket, and does not increase attached gingiva 18. composite is intact but discolored what will u do? Polish and repair 19. picture with multiple periapical radiolucencies ? cemebtoosseous dysplasia 20. differentiate between irreversible pulpits and pulp necrosis? (Necrosis pup is dead pulp and not respons to pulp test, While irreversable pulpitis is vital inflamed pulp and response to pulp testing) 21. half radiolucencies line on ramis ..identify ..fracture , air space ,two options were more ...? Fracture 22. An old patient came with a slowly growing swelling in the angle of the mandible, On radiograph there is radiopaque lesion with radiolucent border. Whats your diagnosis? Ossifying fibroma 23. on radiograph periapical radiolucency is evident? Focal sclerosis osteomyelitis. 24. RG pictures Rg ameloblastic fibro odontoma ? Mixture epithelium and mesenquima, encapsulate. Mandible area common. Radiolucency with opacity 25. Rg of dentigerous cyst? Around unerupted tooth, symetrycal round RL with RP border, starting at the CEJ, associated with the crown of an unerupted teeth 26. very well defined round radiolucency in panoramic, posterior mandible below inferior alveolar canal? static bone cyst () see xary in google

27. X-Ray: Black women, middle aged, anterior radioluceny (can be radio opaque)? cemento osseous dysplasia 28. picture PA of between the trabecula of the bone close to the apex of the teeth? nutrient canal 29. Picture of vermillion border damage on lower lip. And lesion >10cm require? incisional biopsy 30. Probing depth is 5 and prob pass 2 mm apical from CNJ how much is the attachment loss? 7 (5 +2 – remember here it says apical loss over probing depth so we add otherwise we substract) 31. a patient is getting recurrent ulcers in mouth that heal spontaneouly accompanied by a picture, so by seeing a pic one thing you can confirm is that it is aphthous ulcer? bechets syndrome 32. Radiograph to identify the soft tissue? tip of nose along the root tips of centrals 33. Radiograph to identify? external auditory meatus 34. Picture of flared out anteriors asking Class? Class II division I 35. Cross allergy for anesthetics? Know esters and amides 36. radiographic image of (painless lesion, bone expansion)? fibrous displasia (ground glass appearance to identify) 37. Cross sectional studies, Observational study which studies both exposure and disease at the same point of time 38. Clinical trial, Experimental study done after etiology is found to test possible treatment 39. Leukemia picture? bleeding gums 40. management of hypertension? Chlorothiazide as a diuretics for mild cases and Guanethidine for severe cases. (Mild- b blockers and thiazides, Moderate ace inhibitors and alpha blockers, Severe- neuronal depletion agents) 41. opg with a radio lucency extending from post mand to ant? okc (confusing option can be ameloblastoma , but ameloblastoma spreads labiolingually rather than ant post) 42. pano with radiolucency seen with unerrupted tooth? dentigerous cyst 43. On left side max 1 M MB cusp touch man 1 M DB cusp, on right side no max 1M, there is max PM2 touch central fossa on man 1M, and max 2M MB cusp behind man 1M DB cusp (pic). Which class Angle on each side? Class 2 44. Which micro organism in rhomboid glossitis? Pic? Candidiasis 45. Tmj x rays, which xray you do for tmj? MRI 46. X ray - disc of TMJ. 47. Panoramic - auditory meatus 48. PA xray of ant max teeth - soft tissue of nose 49. Pano X-ray 14 yr old pano 30 is rct. Pt complained just enlarge and hyper occlusion? fibrous dysphasia 50. On xray, opacity apical of a RC treated tooth. What could it be? sealer. 51. Elderly pat. With a super old bridge on lower back teeth. Has sensitivity when drinking cold drinks. Which tooth most likely to be the reason? The bridge had 3 abutment teeth. On the x-ray one tooth had a post, another one had a big radiolucency, So, there is only molar, which look normal with a little subgingival calculus? True, Both teeth are non vital, Only tooth with calculus is vital

52. Middle aged woman who wanted all her teeth pulled out, had a radiolucency in the canal of an endo treated upper canine. What could that be? Can be fracture gp condensation or inflammation from coronal pulp (not internal resorption because tooth is endodontically treated) 53. She had multiple fractured teeth, which were all previously endo treated. Why does her oral situation looks like that. All except? external tooth trauma 54. Ameloblastoma x ray? Soap bubble apprearance 55. Guy from Ethiopia had Hep B positive surface antibody- what does this mean? he is fine, he has it and needs vaccine, he has it and needs some medication? He is fine (previous exposure with hep B / must be vaccinated and he is not a carrier) 56. a scenario with a young girl who had a trauma tooth with a “bubble on the gums”, she also drank a ton of soda? Bubble on gum is related to abscess 57. Pic of generalized inflamed gingiva in upper teeth (can be a child) pt was treated for skin disease for 2 month? multiple pyogenic in child 58. Xray of Paget? Cotton woll 59. Radiograph of a remaining root with an apical radiolucency. Remaining root was extracted and the lesion was taken biopsy, results came back with many neutropils. What is the lesion? Cyst? Abscess 60. Lateral ceph with an arrow point on? Mastoid process, coronoid process, ear lobe 61. a pic of buccal mucosa with yellow tiny eruptions a) fordyce granules 2) SCC 3) papilloma? Fordyce granule (buccal mucosa - fat) 62. Radiograph of fibrous dysplasia of jaw day2? Ground glass appearance 63. a pic of teeth, what does the patient has? dentegenous imperfect 64. photo of the teeth, which tooth has low copper amalgam filling? One with discoloration near margin 65. Picture of mucosa in lower lip? Hyperthrofia 66. Picture of this child 11 years old, Maxilary central ok but lateral in crossbite. Sth like? How to treat? maxillary expander 67. Middle aged guy with kidney failure due to Lithium overdose. What pain drug is less expected to be nephrotoxic? Oxycodone 68. Why do we need ruler in lateral cephalogram? For magnification – scale Calibration ruler for magnification correction. Ruler to standardize the magnification rate of radiographs. 69. In removal of palatine tori which structure can be damaged? Greater palatine artery 70. 10 y.o girl, with good OH, no caries but a child of divorced parents. How would you rate her caries risk? Middle 71. 10 y.o child case, where upper canines were closely to errupt, but primary canines were still there. the radiolucency that surrounded the erupting teeth? Eruption cyst. 72. Hispanic guy, no insurance, needs tx. If you extract tooth 14, what is the most expected complication? The tooth had RCT and a very big amalgam fllg? Tooth fracture 73. Pat. Allergic to sulfa, which meds. Are contraindicated? Next antibiotics: Sulfamethoxazole-trimethoprim (Septra, Bactrim), Erythromycin-sulfisoxazole, Sulfasalazine (Azulfidine), used to treat Crohn's disease, ulcerative colitis and rheumatoid arthritis. Dapsone, used to treat leprosy, dermatitis and certain types of pneumonia

74. Middle aged Pat. With an one-tooth gap. If you want to close it orthodontically, what will you expect? No bodily movement of the teeth, Rotation mesio-facially? Rotation mesio- facially 75. two small radioopacities in the bone where MD M1 was missing? Focal idiopathic osteosclerosis. 76. diagnostics of teeth on x-ray, if it is a proximal caries or burnout? If it is on every tooth, it is most likely to be burnout! 77. Test for kidney failure? creatinine 78. RCT of a central maxillary incisor. What is true? it will be difficult compared to a normal case, because canal was really obliterated. 79. Pat. Bites down on his maxillary M1, which already had a super big amalgam filling, and breaks off one of the cusps supragingivally. What tx? PFM 80. On xray, opacity apical of a RC treated tooth. What could it be? sealer. 81. How to treat his posterior cross bite? Hyrax appliance (banded type)—for skeletal expansion, this is the most commonly used type of rapid palatal expansion/rapid maxillary expansion appliance. Haas appliance: However, difficulty in maintaining hygiene and possible inflammation of the palate are considered disadvantages by some clinicians. Hawley-type removable appliance with a jackscrew— for skeletal or dental expansion, this appliance may be used to correct mild posterior crossbites in children and young adolescents. Quadhelix and W-arch—generally for dental expansion, these appliances consist of heavy stainless steel wire with four (quad-helix) or three (W-arch) helices that are incorporated to increase the range and flexibility. 82. On a lateral cephalogramm of the 10 y.o girl what is the radiolopacity that crosses her posterior teeth? Palatine process of maxilla. 83. Hispanic Pat, without insurance. Missing teeth, needs tx. What is the least tx indicated? sinus lift 84. Old pat., heavy smoker with white patch on lip? Hyperkeratosis due to smoking. 85. How to biopsy that hyperkeratosis? Incisional 86. Pat. With leukoplakia on lower side of the tongue. How to biopsy that? - In all cases, leukoplakia must be completely excised since diagnosis cannot be made clinically (DD) 87. What structure can be damaged while biopsying the lower side of the tongue? lingual artery 88. Pat. With HTN. Taking various meds. Long list, Beta-bloker among those. Which ones lowers his BP? Lisinopril Lisinopril: ACE inhibitors, "inhibit" the conversion of inactive Angiotensin I Angiotensin II (a vasoconstrictor). This causes peripheral vasodilation and secondarily increases urinary volume excretion. Both actions cause reduced BP. 89. Pat. Taking antidepresants. Having heavy bruxism. What is least likely to do to treat him? except the one saying to lower his medication 90. Pat. With pigmentation above maxillary lateral incisor. What could that be and how would you biopsy that? excisional, because it was small 91. lderly apt. taking various med. Including Aspirin 81mg. Would you stop Aspirin before SRP? NO 92. Middle aged woman who wanted all her teeth pulled out, had a radiolucency in the canal of an endo treated upper canine. What could that be? Radiolucency in the canal

93. She had multiple fractured teeth, which were all previously endo treated. Why does her oral situation looks like that. All except? external tooth trauma. 94. depressed guy, and his Amitriptyline med.? Tricyclic antidepressants are very likely to cause xerostomia. Amitriptyline is especially potent in this regard. (Mosby) The most common CNS adverse reaction is DROWSINESS. Anti-cholinergic adverse effects are dry mouth (xerostomia), constipation, blurred vision, and tachycardia. Drug-induced xerostomia must be treated palliatively with artificial salivary substitutes. – BB EPI (vasoconstrictor) in local anesthetic injections must be used cautiously in patients taking tricyclic antidepressants (Le. Elavil), Serotonin & NE reuptake inhibitors (Le. Effexor) to avoid transient and significant increases in blood pressure. These antidepressants greatly increase NE levels in tissues. In the presence of a vasoconstrictor administered via a local anesthetic injection, the patient can experience a significant elevation of blood pressure due to the vasopressor actions of the combination. – BB 95. on picture it is obvious Class I relationship of 1st molars (MB cusp of upper 1st molar occludes mesial buccal groove of 1st mand molar) but later there was on cephalometrics pt has ANB=6.? SKELETAL CLASS 2 with dental class 1 (teeth may compensate for dental disharmony) - SKELETAL CLASS 2, DENTAL CLASS 1 96. on picture there is a very deep overbite (upper incisors covered 2/3 of lower incisors), but canines position look Class I (cusp of upper canine occludes in between lower canine and premolar) , and looking at anterior teeth what Class this pt has? Class ll. if anterior looks like cls 2 div 2 along with deep bite +class 1 molar or canine relation termed as DECKBISS 97. On x-ray, there is max lat incisor with RCT done and crown, periapical RL (looks like old RL 4 mm) and within RL RO in the middle... what is that? Hypercementosis 98. Same Pt had this max upper lat inc and 1 st molar only left on that side. Pt Decided to do RPD. 1) Because of the absence of canine it affects this lat inc long prognosis? True 2) Because this upper denture has no vertical stop with mandible on that side it affects this upper lat inc long prognosis.True 99. pt has mild crowding in lower ant teeth, narrow arch, no crowding in maxilla, but narrow arch, very little overbite (like 1mm), long face. she expected to have all, except? Obtuse nasolabial angle (it can’t be obtuse angle because of the flarring of teeth) 100. Pt has mand premolar and edentulous posterior to it. What you won’t place on it restoring the space? distal rest with a buccal retentive clasp 101. You are doing a wheel chair transfer, which of the following will you not do? not to use the belt of the pt. 102. Pregnant pt, When is the best time to treat her? Best time 2 nd trimester during pregnancy. 103. What meds can u give her or not? acetaminophen you can. 104. What is most likely to lead her present with a syncope? pressure on IVC 105. Old pt, 84 yo, what is your primary concern? involve a guardian in his tx plans 106. Stubborn pt, comes in coz of dissatisfaction of his previous several dentists. Despite of so many changes he still seeks for better prosthesis, what makes u determine if he will be satisfied by your tx or not? the most affirmative and agreeing to what the pt was saying, to build rapport and form of trust is most imp 107. Pt on anti hypertensive drugs, anti diabetic drugs, some numerical readings, looked normal? proceed with the tx

108. Pt on no significant med history, past history of some surgeries, blood pressure comes out to be a little high, what will you do next? measure BP after 15 mins and see what comes up 109. Pt has AIDS, his viral load is 1,000, T-cell count is 30, what will be your inference? his T-cell count is too low and that might put him at a risk of infection 110. AIDS pt in first stage. Which of the following will you notice? he is symptomless in this stage 111. Pt on coumarin, what lab test will you look for? PT 112. Malignancies to the oral cavity from the rest of the body are most likey to end up in which region? floor of the mouth 113. Which equipments in the dental health clinic will need a signed something? Lasers 114. Pt comes in with signs of physical assault, whom shall u contact first? talk to pts guardian 115. Denture wearing pt has pink raised lesions on the palate, what can it be? papillary hyperplasia 116. Tori on upper palate, extending to the post palatal seal, what decides whether you remove it or not? Undercuts affecting the retention of the denture 117. Ortho forces applied, what is least likely to happen? Differentiation of cells due to chemical influences in the pdl (Application of orthodontic forces will provoke a haemodynamic response in the pulp). 118. Pt comes to you with a catheter, which fo the following will facilitate your tx? leave it as it is and take detailed history again 119. Drugs not to give to a pregnant lady? NO 120. Pt only visit dental office for emergency treatment, all will help him to change his behaviour into routine check ups instead of emergency visits only by dentist? Except positive reinforcement (Reinforcement means behavior increases, we don’t want that.)

121. Middle aged guy with kindney failure due to Lithium overdose. What pain drug is less expected to be nephrotoxic? Aspirin, Ibuprophen, Oxycodone, one more? Acetaminophen (if not in option, ibuprofen) (also in kidney failure we can give tramadol, if not in option, oxycodon) 122. Why do we need ruler in lateral cephalogram? For magnification (if not in option, go with measurement, reference carranza) 123. In removal of palatine tori which structure can be damaged? Greater palatine nerve and artery 124. 10 y.o girl, with good OH, no caries but a child of divorced parents. How would you rate her caries risk? Low, Middle, High? Low (child knows to take care of her OH) (Debbatable with middle) 125. Hispanic guy, no insurance, needs tx. If you extract tooth 14, what is the most expected complication? The tooth had RCT and a very big amalgam fllg. Sinus perforation, Ridge fracture, Tooth fracture, Bleeding? Tooth fracture 126. Q. about that 10 y.o child case, where upper canines were closely to errupt, but primary canines were still there. They asked about the radiolucency that surrounded the errupting teeth. Options were different kinds of cysts and tumors.? Dentigerous cyst (if not in option, go for eruption cyst) 127. Pat. Allergic to sulfa, which meds. Are contraindicated? Sulfonamides, bactrim 128. Middle aged Pat. With an one-tooth gap. If you want to close it orthodontically, what will you expect? No bodily movement of the teeth, Rotation mesio-facially, Rotation mesio-lingually, one more? Mesiofacially 129. A Q. about two small radioopacities in the bone where MD M1 was missing. There were options about Odontogenic tumors, Root rests, Focal idiopathic osteosclerosis.? Focal idiopathic osteosclerosis. 130. Q on that child case which bite relationship it has. Distal, Mesial, End-to-end 131. Test for kidney failure. Creatinin 132. Q on except for the 3rd molard, how many teeth was the child missing. 133. Q on RCT of a central maxillary incisor. What is true? it will be difficult compared to a normal case, because canal was really obliterated. 134. Pat. Bites down on his maxillary M1, which already had a super big amalgam fllg, and breaks off one of the cusps supragingivally. What tx. ? PFM Crown (not onlay in this case) 135. On xray, opacity apical of a RC treated tooth. What could it be? Looked like sealer. 136. Pat. Has very strange bite, posterior crossbite, Class 2 relationship on the molars, but almost perfect overbite. What is the most likely cause? chose something with the Canine 137. How to treat his posterior cross bite? Quad helix expander 138. Female pat. Wants all her teeth extracted, although they don't look as bad. Qs. About what tx. Is appropriate.? Patient education 139. That 10 y.o girl had a very strange bite, with a skeletal midline deviation to the left. They ask about the cause. Can be TMJ, condylar hyperplasia or bone tumor 140. On a lateral cephalogramm of the 10 y.o girl what is the radiolopacity that crosses her posterior teeth? Maxillary sinus, Orbita, Zygomatic arch, Palatine process of maxilla? Zygomatic arch 141. Hispanic Pat, without insurance. Missing teeth, needs tx. What is the least tx indicated? sinus lift 142. Qs about that 10 y.o girl who was asmathic. Meds.? Albetrol 143. Old pat., heavy smoker with white patch on lip? Hyperkeratosis due to smoking. (Can be leukoplakia too coz patient is smoker. Depends on exact case) 144. How to biopsy that hyperkeratosis? Incisional 145. Pat. With leukoplakia on lower side of the tongue. How to biopsy that? Exfoliative cytology 146. What structure can be damaged while biopsying the lower side of the tongue? Lingual nerve, mylohyoid muscle 147. Pat. With HTN. Taking various meds. Long list. Lisinopril, Beta-bloker among those. Which ones lowers his BP.? Lisinopril 148. Easy Q on tx. Of a RCT tooth with a big amalgam fllg. 149. Elderly pat. With a super old bridge on lower back teeth. Has sensitivity when drinking cold drinks. Whisch tooth most likely to be the reason? The bridge had 3 abutment teeth. On the x-ray one toothe had a post, another one had a big radiolucency and to me it looked dead! :-D So, it was the only molar, which looked normal to me with a little subgingival calculus. So, I picked that!? True, Both teeth are non vital, Only tooth with calcilus is vital 150. Pat. Taking antidepresants. Having heavy bruxism. What is least likely to do to treat him. All options were correct, except the one saying to lower his medication!? lower his medication 151. Pat. With pigmentation above maxillary lateral incisor. What could that be and how would you biopsy that? Incisional biopsy (is small lesion) 152. A Q. about that elderly apt. taking various med. Including Aspirin 81mg. Would you stop Aspirin before SRP? No, doesn’t need too stop baby aspirin 153. Middle aged woman who wanted all her teeth pulled out, had a radiolucency in the canal of an endo treated upper canine. What could that be? Can be fracture gp condensation or inflammation from coronal pulp (not internal resorption because tooth is endodontically treated) 154. Another Q about her, showing a lateral upper incisor with post and crown, asking how that post looks on the x-ray. Too narrow, too long, too wide, too short 155. Another Q about her. She had multiple fractured teeth, which were all previously endo treated. Why does her oral situation looks like that. All except question. I took external tooth trauma? external tooth trauma 156. Q. about that depressed guy? Amitriptyline med 157. Pt has all canines erupted ectopic, but the rest of occlusion looks fine, what should be done? extract premolars and with fixed ortho move canines on their place 158. If she decided to extract premolars what forceps not to used? 23 159. q about the profile? It was convex depending in the photograph 160. what is the preventive treatment for this patient? encourage the patient to use brushing and dental floss 161. What is the most costly to do to prevent declassification around the braces? varnish every 6 month 162. what is the treatment of choice for tooth #19 if it is already catch the prob during examination? compsite filling 163. a patient broke his tooth – max 1 pm. He had excellent hygiene. Tooth broke 2 mm under the gingiva. It has big amalgam filling. What you do? RCT and post and crown. 164. Case with the man with allergy to penicyline and clindamycine..He had a joint Replacement? He doesn’t need prophy for that 165. if he need prophy for treatment what you give him? Azitomycin 166. He has terrible pain in the face. During the night too. It lasts 5 sec. It comes and go away. What is it? Neuralgia 167. Case with 5 years old girl She had all first molars and erupting max central. Second central incisors – had accident and was avulsed. What you do with concussion? observe and control max central 168. her age. Is her dental age? the same as bone age 169. She had dark changes in her gingival? racial changes sth like this 170. on her gingiva above her central incisor.? It was sinus tract Because of her accident 3 mouth earlier. 171. She has lateral crossbite – how to treat it.? maxillary expansion

172. Can you give her for daily use CHX – T/F? True 173. You should show her and her parent how to brush T/F? True 174. Man with hyperlipidemia. Triglyceride is high. What do you suspect? Metabolic syndrome. 175. Metabolic syndrome can cause cancer of all except one? Thyroid 176. Treatment of this patient? Initial therapy (OHIs – SRP – Caries control – crown lengthening) 177. Man smoking. He had also rheumatoid disease. He has hypertension. Tekes Propranolol. Is taking Chantix. What he can have? Xerostomia 178. Which drug causes dry mouth? Chantix 179. You give him LA with epi, what happened? blood pressure elevated 180. It doesn’t work – what else can you give him? a. bupropion Zyban b. nasal spray c. nicotine gum? Nicotine gum 181. Which ASA classification he is? class II. RA (ASA classification) 182. Sequence of tx plan? (emergency – caries control – reevalute – definitive treatment – maintenance ) ECRDM 183. Children with cleft palte? Pano and cephalo. Given SNB - 73 and ANB -2 what angle class is it.? class III malocclusion. 184. Patient had lateral crossbite – why? maxilla to small and lateral shift to the right side. 185. What to do with teeth S – teeth was ok? don’t treat but observe 186. Profile of this child in cephalo How many teeth is missing, Had also some supranumerary? count them. 187. Can’t implant in? cleft palate 188. Tooth is painful, mand first molar. What to do? Extract 189. Obese man with hypertension - 190/110 Didn’t treat it. Haven’t been in doctor 10 years. No medicaments. He is going to the restroom 2 per night, What do you suspect? diabetes type II 190. What is the first think you will do to treat this patient? Refer to the physicans 191. What contribute to the future risk of caries in this patient, exept one, which one? many sweets and bad diet (AMOUNT) 192. Child with ADHD, What is he taking? amphetamine 193. He can’t sleep and eat? because of this treatment 194. He had insomnia, why? amphetamine (indirect acting symphatomimetic) Not to many things during one appointment. 195. Don’t use tell show do? False, we use it. 196. ADHD is most common with? males 197. if you can give child with ampfo Or should you lower the dose of amphetamine? No need to adjust dose 198. case about 8yr 7 months child with supernumerary anterior teeth. Patient had class 2 skeletal and class dental malocclusion due to? missing canine teeth. 199. SNA was 87 and SNB 82- what will we call? Maxillary protrusiom 200. patient had Hep A 20 years ago what lab test you need? Nothing. We don't care about his hep A 20 years ago

DAY 2 1. 5 yrs girl lost upper second primary M – space maintainer? – distal shoe 2. HSV –valcyclovir 3. C factor 4. For increasing support for lower denture? – lingual plate, buccal shelf, clasp, .. 5. Max sin antibiotic – amoxy + clavulanic acid 6. Zoloft SE? _xerostomia? 7. Zyrtec SE? – xerostomia? 8. Maslow theory? 9. Angle class III, I 10. Papilloma 11. RO line on X –ray upder filling – Ca(OH) base, GI liner? 12. 13. Percussion+ - acute apical abscess, symptomatic apical periodontit 14. 9 yrs girl –dental age and her age – early, late 15. Convex profile 16. Lateral incisors –antrior cross bite: why? – max crawding 17. Tooth movement - tipping 18. Not enough space for lateral icisors – extract primary canine 19. Lower incisors –retrocline, 20. Lower incisors - thin gingive, NO – traumatic occlusion 21. Lost early primay first molar- decreased space why? - Mesial movement primary m2, permanent m1 T/F - Distal drift primary canine T/F

22. Lower midline shifted to the left why? - Early eruption lower second icisors 23. Mesial root of mandible second primary molar resorption on x-ray? Normal Replacement Inflammatory Infection

Case 2: Male 64 yrs with a white lesion on lip Smoke cigarettes for 30+ years and also smokeless tobacco Bad Xray Pics so hard to see whats really going on in the mouth

1)What is in the microscope of brown pigmentation? Hemosiderin Melanin Melanocytes Smth like blood vessels

2)The reason of that brown pigmentation Smokeless tobacco Smoking cigarettes Medication Smth else

3)The reason of that white lesion on the lip Smokeless tobacco Smoking cigarettes Medication Smth else

4) Some big yellow nodule on the buccal (looked like ) but yellow and q was about one nodule and it was inside the buccal… quality of the picture was bad…couldn.see SPOTS clearly What is inside? Adipose tissue I put this dunno why. don't ask. Lymphoepithelial cyst Mucous glands Sebaceous glands this is for Fordyce spots

5)Biopsy Should include everything except Normal site or smth Submucous tissue Smth else

6)Biopsy question on the ventral side of the tongue EXCEPT Be careful submandibular gland Be careful to avoid sublingual gland